Criminal Law In-Class Questions

Ace your homework & exams now with Quizwiz!

Stanley is charged with arson. The crime of arson requires that a defendant maliciously set fire and cause damage to property. Stanley's house caught fire when he was setting off fireworks in his yard. Part of the house burned before firefighters could extinguish the fire. What would the prosecution have to prove for Stanley to be guilty of arson? A. Stanley knew he could set his house on fire but ignored the risk and set off fireworks too close to his home. B. Stanley's purpose in setting off the fireworks was to burn down his home. C. Stanley always hated his home and therefore wanted to burn it down, so he set off the fireworks. D. Stanley carelessly burned down his home.

A. Stanley knew he could set his house on fire but ignored the risk and set off fireworks too close to his home. Answer A is the best answer because the minimum intent required for most crimes is the standard of recklessly or maliciously. In those crimes, the defendant must actually consider the risk that the property would be burned and disregard that risk. Here, Answer A provides that scenario. By contrast, when a defendant acts in an unthinking manner, the defendant may act negligently or carelessly. Answer B is wrong because the standard of maliciously does not require this high level of intent. Answer C is wrong because mens rea refers to a defendant's intent, not motive. Maliciously, does not require that Stanley had an evil motive to burn down his home. In other words, his motive is irrelevant, if he acted with the requisite intent. Answer D is wrong because the legal standard of maliciously requires that a defendant act more than just carelessly.

Calvin is a chronic alcoholic. Seldom does a day pass when he doesn't end up drunk by lunchtime. As a result of his problem, Calvin has had some encounters with law enforcement. In particular, he is facing charges for breaking a beer bottle over a security officer's head, willfully evading his taxes, and driving without a license. For which of these crimes, if any, is Calvin likely to be able to argue an intoxication defense? A. Willful tax evasion. B. Assaulting the security officer. C. Driving without a license. D. All of the above.

A. Willful tax evasion. Answer A is the best answer because willful tax evasion requires specific intent. Willfully as a culpable intent would require at least knowingly or intentionally. In determining whether an intoxication defense is available, one must determine whether the charge is a general or a specific intent crime. The defense is available only to specific intent crimes. Answer B is wrong because assault does not require specific intent. Answer C is wrong because driving without a license is generally a strict-liability crime. Answer D is wrong because the fact that assault is a general intent crime and driving without a license is generally a strict-liability crime means that the intoxication defense was not available to Calvin for these charges. Consequently, all of the above cannot be correct.

Reese has been despondent. Nothing has been going right in her life. She decides to take her own life by jumping off a bridge. Renee drives to a bridge, climbs on a railing, and says her final words. However, before she can jump, defendant Jones shoots her. Renee falls to her death. The coroner cannot get a definitive answer as to whether the main cause of Renee's death was the fall or the gunshot. Can Jones be prosecuted for murder? A. Yes, Jones was a cause in fact (but for cause) of Renee's death. B. No, Renee caused her own death. C. No, Renee would have died anyway, even if Jones had never shot her. D. Yes, Jones had the intent to kill Renee.

A. Yes, Jones was a cause in fact (but for cause) of Renee's death. Answer A is the best answer because although the gunshot may not have been the only cause of Renee's death, it was a cause in fact. In other words, but for Jones shooting at her, one cannot state with any certainty that Renee would have plunged to her death at that moment. Even with other causes, the actions by Jones are sufficient to establish an actual cause. Answer B is wrong because it is factually incorrect to state that Renee caused her own death. She may have been about to kill herself, but, at that precise moment, it was the shot that led to her death. Answer C is wrong because it argues a point that is unknowable. Renee could have decided to forgo jumping and gotten down. A defendant's act need not be the sole cause of the death. Answer D is wrong because mens rea and actual causation are separate elements. In other words, the prosecutor would have to prove both causation and intent or there can be no conviction. See People v. Acosta, 284 Cal. Rptr. 117 (Cal. App. 1991) (finding that there was both actual causation and legal causation, but overturning the conviction because the defendant lacked intent).

Falcone sells a sugary substance called lactose. Lactose can be used to supplement baby formula or as a cutting agent to dilute drugs. Approximately 75 percent of Falcone's sales are to Gordy who, as Falcone knows, uses the lactose to dilute and package illegal drugs. Falcone knows he can charge Gordy a little more than other customers because he isn't likely to complain to the authorities. Thus, over the course of a year, Falcone makes numerous sales of large quantities of lactose to Gordy. Gordy and Falcone are charged with conspiracy to distribute drugs and distribution of drugs. Is Falcone guilty of the conspiracy? A. Yes, because he had the necessary mens rea for conspiracy. B. No, because it is not per se illegal to possess or sell lactose. C. Yes, because he was reckless in selling lactose to a drug dealer. D. No, because he was in an official partnership with Gordy.

A. Yes, because he had the necessary mens rea for conspiracy. Answer A is the best answer because Falcone has committed to the conspiracy when he crossed into having a stake in Gordy's venture, enabling prosecutors to argue that he has the intent for his lactose to be used for illegal purposes. Answer B is wrong because even the sale of a lawful product can create a stake in an illegal venture, although it requires more of a showing of purposefulness. However, there is no requirement that a co-conspirator provide something that is illegal in all settings. Answer C is wrong because the standard, even in these circumstances, is not mere recklessness. The ordinary standard for conspiracy is purposefulness. Even in jurisdictions that reduce the standard, it does not fall lower than knowingly. Answer D is wrong because there is no requirement that co-conspirators be in a formal partnership. By its nature, criminal activity involves clandestine activities. The legal status of the partnership is not important. The focus is instead on the defendant's intent.

Oswald has decided to kill the Vice President. He buys a high-powered rifle, sets up an ambush spot and waits for the Vice President's motorcade to pass. Oswald has his rifle beside him and is prepared to pick it up at any moment. Less than a minute before the motorcade passes, Oswald is interrupted by a cellular phone call from his mother reminding him to pickup a loaf of bread from the market. Because he is listening to his mother's message, Oswald does not have an opportunity to shoot and kill the Vice President. Under the Model Penal Code approach, is Oswald guilty of attempted murder? A. Yes, because he took a substantial step strongly corroborative of his intent to kill. B. No, because Oswald was not in dangerous proximity of killing the Vice President. C. Yes, because Oswald had already taken the last step toward killing the Vice President when his phone rang. D. No, because Oswald's actions did not show his unequivocal intent to kill.

A. Yes, because he took a substantial step strongly corroborative of his intent to kill. Answer A is the best answer because correctly states the Model Penal Code standard of requiring a substantial step. Here, Oswald was lying in wait with a gun. Answer B is wrong because it does not rely on the Model Penal Code standard. Moreover, it is probably factually incorrect to maintain that Oswald was not in dangerous proximity of killing the Vice President. Oswald was within shooting range and had demonstrate enough reason for law enforcement to get involved. Answer C is wrong because the Model Penal Code does not require that the defendant take the last step toward completing the crime. All that is required is a "substantial step," which Oswald met when he got a gun, found a hiding place, and planned out the shooting. Answer D is wrong because it uses the wrong standard as unequivocality is one of the many common law standards. Pursuant to that test, an argument could be made that Oswald's action did not unequivocally demonstrate his intent to kill because if he had such an intent, he never would have answered his phone. However, pursuant to the Model Penal Code, his actions in general were generally strong enough to corroborate an intent to kill.

Julia broke into the home of her neighbor, an old enemy, and was going to shoot him while he was sleeping. Julia was unaware that her husband had earlier unloaded her gun, for fear that she would do something rash and kill someone. The neighbor awoke and saw Julia, so she ran out of his house. Can Julia be convicted of attempted murder even though the gun was unloaded? A. Yes, the fact that the gun was unloaded does not preclude a conviction of attempt. B. Yes, because Julia broke into the neighbor's house, she must have meant to kill him. C. No, because Julia may have only wanted to scare the neighbor. D. No, because there was no way under these circumstance that Julia could have killed the victim.

A. Yes, the fact that the gun was unloaded does not preclude a conviction of attempt. Answer A is the best answer because this is a classic scenario of factual impossibility doctrine, which is not a valid defense to an attempt crime. There would be no justification to allow a defendant to escape criminal liability because fortuity prevented the commission of the offense. Julia acted in a decisive manner intending to kill her neighbor and had she been successful she would had violated the law. Answer B is wrong because this response does not truly deal with the impossibility issue. Answer C is wrong because this response ignores the impossibility issue as Julia took every act possible to ensure the killing would occur had it not been prevented by her own stupidity or mistake. Answer D is wrong because pursuant to the factual impossibility doctrine, a defendant can be found guilty regardless of whether ultimate success was a real possibility.

Arthur and Kirk are at a cocktail party when Kirk says to Arthur, "the time has come for me to kick your butt." Arthur starts to laugh and says, "You shrimp. There's no chance you can hurt me." As Kirk looks the other way, Arthur kicks him in the rear. If Arthur is charged with assault and he claims self-defense, his defense will likely A. fail because he did not fear Kirk. B. succeed because Kirk threatened him first. C. succeed because he honestly feared Kirk. D. fail because Kirk did not threaten imminent harm.

A. fail because he did not fear Kirk. Answer A is the best answer because if Arthur did not fear Kirk, then he is not entitled to claim self-defense. Answer B is wrong because it does not matter that Kirk threatened Arthur first. If that threat did not cause Arthur any fear, then he cannot claim self-defense. Indeed, the facts clearly establish that Arthur is not afraid of Kirk. In fact, he was bent on humiliating him. Answer C is wrong because the facts clearly establish that Arthur is not afraid of Kirk. In fact, he was bent on humiliating him. Answer D is wrong because factually Kirk did threaten Arthur with any imminent harm, but the problem for Arthur is that he did not fear the threat.

Andre hates to lose. For the last five years, he has enjoyed the spotlight as the world's greatest chess player and as a wonderful humanitarian an philanthropist. However, there is a new chess competitor coming up in the ranks quickly. The new whiz kid is Nicolai. He is now the darling of the chess world and he has been bragging that he is better and smarter than Andre. Andre tells a friend that Nicolai's days are numbered. The next day, he shoots Nicolai in the head during a tournament. As Nicolai collapses, Andre simply smiles and says, "checkmate, game over." Andre is charged with murder. He claims that he did not act with malice. He is likely to be found A. guilty because he had the intent to kill Nicolai. B. guilty because he had a motive to kill Nicolai. C. not guilty because the prosecution's case is based on circumstantial evidence. D. not guilty because Andre is a man with a good and generous heart.

A. guilty because he had the intent to kill Nicolai. Answer A is the best answer because all the evidence points to the conclusion that Andre intended to kill. Answer B is wrong because it only focuses on motive, which may be used to prove intent to kill, but does not fully answer the question of whether Andre acted with malice. Answer C is wrong because it is incorrectly implies that a prosecution cannot succeed in proving malice with circumstantial evidence. Answer D is wrong because even a person who is generally good and generous can act with malice if he intentionally kills another person. In other words, even if Andre is the nicest person in the world, if he intentionally killed Nicolai, then he acted with malice.

Melinda is a convicted felon who is required by state law to register with the county sheriff where she lives. A sheriffs deputy encounters Melinda and arrests her for failing to register as a convicted felon. Melinda tells her court-appointed attorney that law enforcement is just hassling her because she is a convicted felon. Should Melinda be charged with this offense? A: Yes, because Melinda is being charged for her conduct as opposed to her status as a convicted felon. B: Yes, because Melinda's status as a convicted felon is not protected. C: No, because Melinda cannot be charged based on her status as convicted felon. D: No, because Melinda, as a convicted felon, still has a constitutional right to live in any state she chooses.

A: Yes, because Melinda is being charged for her conduct as opposed to her status as a convicted felon. Rationale: Answer A is the best answer because Melinda is being charged for her conduct of failing to comply with state law by complying with the registration statute as opposed to her status as a felon. States can criminalize conduct even if it most likely affect only people of a certain status. See Powell v. Texas, 392 U.S. 514 (1968). Answer B is incorrect because Melinda is not being charged for her status as a felon, which would be improper, see Robinson v. California, 370 U.S. 660 (1962), but was charged for her conduct, i.e., failure to register, that was illegal. See Powell v. Texas, 392 U.S. 514 (1968). Answer C is incorrect because Melinda is not being charged for her status as a felon, but for her conduct of failing to register. Answer D is incorrect because Melinda is engaged in more than just being a felon. However, Melinda cannot constitutionally be charged for her status as a convicted felon. See Robinson v. California, 370 U.S. 660 (1962).

Rick and Steve met at a bar and decided to rob a bank. Rick bought guns and masks for both of them. Steve cased the bank to learn when the best time of day to rob it was. On the way to the bank, they get pulled over and the police officer notices the guns and masks before placing them under arrest. Are Rick and Steve guilty of conspiracy? A: Yes, because they both agreed to smuggle the aliens and there was a least one overt act in furtherance of that conspiracy. B: Yes, because they have taken a substantial step toward robbing the bank. C: No, because they have not taken a substantial step toward robbing the bank. D: No, because it was not illegal for Rick to purchase guns or masks.

A: Yes, because they both agreed to smuggle the aliens and there was a least one overt act in furtherance of that conspiracy. Rationale: Answer A is the best answer because both defendants had all the necessary elements for the crime of conspiracy, including the simple overt act of checking the weather report. Answer B is wrong because substantial step is the standard for an attempt. Answer C is wrong because substantial step is the standard for an attempt. For an overt act, any act to further the conspiracy would qualify. Answer D is wrong because it is not required that the overt act be illegal in itself. It can be a totally lawful act and still quality as an overt act.

Hank sees an unconscious drunk on the sidewalk and cautiously approaches him. Hank can see that the man has a wallet in his pocket. Hank carefully reaches into the victim's pocket and pulls out his wallet. Hank checks the wallet for identification, but when he finds none, he puts the wallet in his own pocket and walks away. Although Hank is carrying a concealed weapon at the time of the encounter, he never displays it while he is taking the wallet. Hank is apprehended shortly thereafter. Which crime(s), if any, is Hank guilty of? A. Robbery. B. Larceny. C. Robbery and larceny. D. No crime.

B. Larceny. Answer B is the best answer because Hank did not use force or threat of force to take the drunk's property. Even though he has a weapon, he did not display it or threaten any force. Thus, he cannot be found guilty of robbery. However, Hank's action meet the elements of larceny as he took the property of another with the intent to permanently deprive him of that property. Thus, he can be found guilty of larceny. Thus, Answers A, C, and D cannot be correct.

When he was young, Michael was repeatedly beaten by a man who always wore a blue tie. The beatings were severe and Michael almost died from one of them. One day when Michael was an adult, a man with a blue tie accosted Michael on the street, yelling at him and clenching his fist. Michael responded by pulling out a gun and killing the man. Michael was charged with murder. Assuming we are in a jurisdiction that recognizes imperfect self-defense, which of the following is correct? A. Michael is not guilty of any crime. B. Michael is guilty of manslaughter, not murder. C. Michael is guilty of murder because he intended to kill the victim. D. Michael is guilty of murder if he was negligent in shooting his victim.

B. Michael is guilty of manslaughter, not murder. Answer B is the best answer because a successful imperfect self-defense reduces the charge from murder to manslaughter. Answer A is wrong because no jurisdiction adopts a purely subjective approach to self-defense. To do so would give paranoid defendants a license to kill. Answer C is wrong because an imperfect self-defense claim reduces the charge from murder to manslaughter, and this jurisdiction recognizes imperfect self-defense. Answer D is wrong because if a person is negligent, he is not guilty of murder. The applicable charge is involuntary manslaughter or negligent homicide.

When he was young, Michael was repeatedly beaten by a man who always wore a blue tie. The beatings were severe and Michael almost died from one of them. One day when Michael was an adult, a man with a blue tie accosted Michael on the street, yelling at him and clenching his fist. Michael responded by pulling out a gun and killing the man. Michael was charged with murder. Assuming we are in a jurisdiction that recognizes imperfect self-defense, which of the following is correct? A. Michael is not guilty of any crime. B. Michael is guilty of manslaughter, not murder. C. Michael is guilty of murder because he intended to kill the victim. D. Michael is guilty of murder if he was negligent in shooting his victim.

B. Michael is guilty of manslaughter, not murder. Answer B is the best answer because a successful imperfect self-defense reduces the charge from murder to manslaugther. Answer A is wrong because no jurisdiction adopts a purely subjective approach to self-defense. To do so would give paranoid defendants a license to kill. Answer C is wrong because an imperfect self-defense claim reduces the charge from murder to manslaughter, and this jurisdiction recognizes imperfect self-defense. Answer D is wrong because if a person is negligent, he is not guilty of murder. The applicable charge is involuntary manslaughter or negligent homicide. See Laurie Levenson, Glannon Guide to Criminal Law

Mickey is charged with "knowingly using the credit card of another person." The charges arose when Mickey stole the wallets of a couple of customers shopping at an upscale department store. Upon reflection, Mickey felt badly about stealing the credit cards. Therefore, he decided that he would only charge items on the credit card of the customer who looked like she could afford it the most. While on his shopping spree, Mickey accidentally gave the sales clerk the credit card of the less-affluent customer. Does Mickey have a mistake of fact defense? A. Yes, because he honestly intended to use the credit card of a different customer. B. No, because he violated the charged statute. C. Yes, because he would not have used the credit card if he knew it belonged to the less affluent customer. D. No, because mistake of fact is never a defense to a common law crime.

B. No, because he violated the charged statute. Answer B is the best answer because Mickey has a mistake that does not matter. Specifically, he knows that he is using another person's credit card so that it does not matter if he correctly identifies that person. In other words, even with his mistake he still would have violated the statute. Answer A is wrong because even if Mickey made an honest mistake it does not matter as he knowingly used another person's credit card in violation of the statute. Answer C is wrong because as long as Mickey knowingly used another person's credit card, he has violated the statute. Thus, Mickey's motive is irrelevant as motive is not an element of the crime. Answer D is wrong because the mistake of fact defense can be used in some common law crimes. The use of words like never or always in an answer typically will be a trigger that the answer is incorrect.

Sally found out, through the sue of a private investigator, that her husband had been having an affair with Kate, Sally's best friend since childhood. Furious with Kate and her husband, Sally decided to get back at both of them. Her plan was to tamper with her husband's car before he left work to go "bowling" (which, she had found out, was actually his cover-up for the times he went with Kate to a local motel), so that he and Kate would be involved in a fiery crash. However, Sally knew nothing about cars. In order to prepare for her plan, she took a basic auto class through the local community college. She did not tell her husband about the class. One day she returned from class and found her husband holding up a tuition bill and a bill for the private investigator. Sally dissolved into tears and told him that she knew about the affair; she also told about her plans to kill him and Kate. Sally's husband recorded the entire conversation and turned the recording over to the police, along with the two bills. Can Sally be found guilty of attempted murder? A. No. It is not clear that Sally really intended to murder her husband and Kate. B. No. Sally had not yet completed the auto class and had plenty of time left to change her mind. C. Yes. Sally had taken a substantial step towards her plan to kill the two by enrolling in the auto class. D. Yes. Sally had a plan and took action by taking the auto class. In addition, there is no evidence to suggest that she would have aborted the plan if her husband had not confronted her with the two bills.

B. No. Sally had not yet completed the auto class and had plenty of time left to change her mind. Answer B is the best answer because the attempt offense requires both the intent to commit the crime and some sort of substantial act in furtherance of the crime. Although Sally's intent to murder her husband and Kate may be clear from her enrollment in the auto class and her confessed plan, she had not yet tried to implement the plan. Action toward the crime is required for attempt because the law does not penalize bad thoughts alone. It could be argued that enrolling in the auto class was sufficient action toward implementation of the crime, but this is unlikely to be found to be a substantial enough step criminal liability. Sally had many more acts to still complete and time to reconsider and abandon her plan. Answer A is wrong because Sally's confession demonstrated her intent. The problem here relates to whether the act would be considered beyond mere preparation, not whether she had the requisite mental state. Answer C is wrong because enrollment in the class would not be a substantial enough act in furtherance of the crime. Generally, more than the merest preparation toward the commission of the murder is required. Sally's enrollment would be too minor an act to qualify for an attempt conviction. Answer D is wrong because the only relevant inquiry is whether the act taken is mere preparation.

Sally, on her way home from finding out from the private investigator that her husband was having an affair, was so upset that she ran a red light going 75 MPH in a 25 MPH speed zone. By pure coincidence, the car she crashed into was her husband's car. He was severely injured, but ultimately made a miraculous recovery. Can Sally be convicted of the crime of attempted involuntary manslaughter? A. No. Running a red light, even at such a high speed, is not reckless enough for a conviction of either involuntary manslaughter or the attempt offense. B. No. Sally lacks the requisite mens rea for the charge. C. Yes. Sally's actions would have constituted involuntary manslaughter had her husband died from his injuries. D. Yes. Sally must have subconsciously intended to hit her husband, so the intent required for intent is met.

B. No. Sally lacks the requisite mens rea for the charge. Answer B is the best answer because Sally may have acted recklessly, or with knowledge of the ramifications of her actions, allowing for a conviction of the crime of involuntary manslaughter had her husband died, he did not die. While involuntary manslaughter requires a mens rea of just recklessly, attempt requires a higher state of mind. Here, Sally's mens rea would not allow for a finding of attempt because she did not intend a killing. Thus, she cannot be convicted of attempted involuntary manslaughter. The crime of attempt requires intent. Answer A is wrong because running a red light at such a high speed would likely be found to be sufficiently reckless in violation of the law. Answer C and D are wrong because while involuntary manslaughter requires a mens rea of just recklessly, attempt requires a higher state of mind. Here, Sally's mens rea would not allow for a finding of attempt because she did not intend a killing. Thus, she cannot be convicted of attempted involuntary manslaughter.

Ralph has always had a crush on Ellie, but she has never responded to his advances. Even though he knows she will be furious, Ralph goes ahead and give her a kiss as they are riding in a crowded elevator. Ellie reports him to the police and Ralph is charged with battery. Which of the following is correct? A. Ralph is not guilty unless he hurt Ellie with his kiss. B. Ralph is guilty because he touched Ellie in an offensive manner. C. Ralph is not guilty because he never assaulted Ellie. D. Ralph is guilty because others in the elevator saw him kiss Ellie.

B. Ralph is guilty because he touched Ellie in an offensive manner. Answer B is the best answer because Ralph's kiss constituted an offensive touching, which is sufficient to establish a battery. Answer A is wrong because a battery does not require an injury. Answer C is wrong because assault and battery are two different offense. Assault is an attempted battery. However, once the unlawful touching has occurred, the defendant is guilty of battery. Answer D is wrong because Ralph's kiss would have been offensive to Ellie even if other did not witness it.

A child darted out into the road in front of Ellie, who was driving very fast. Although Ellie applied the brakes, she still hit the child and caused serious injuries. Fortunately, the child did not die as a result of the accident. What crime, ... , might Ellie be found guilty of due to the accident? A. Attempted murder. B. Reckless endangerment. C. Manslaughter. D. Negligent homicide.

B. Reckless endangerment Answer B is the best answer because reckless endangerment statutes have been enacted in many jurisdictions to criminalize a defendant's reckless behavior that causes serious injury short of a fatality. Answer A is wrong because this charge would likely be unsuccessful. Although Ellie's actions threatened the child's life, she did not act with intent to harm the child. Attempt crimes always require that the defendant acted purposefully or with intent. Answer C is wrong because the child did not die. Manslaughter would have been a viable option if the child died. Answer D is wrong again because the child did not die. Negligent homicide also would have been a viable option if the child died.

Rodney lives in a mansion with a fabulous art collection. He uses two pit bulls to protect his home. When Rodney is on vacation, Charlie tries to break into the home to steal one of Rodney's priceless Monets. Charlie is mauled to death by the pit bulls; Rodney is charged with homicide. Which of the following is incorrect? A. Pit bulls are like spring guns—they cannot be used to protect property alone. B. Rodney could use the pit bulls because he has a right to protect his possessions. C. Rodney could have used the pit bulls if he had been home when the break-in occurred. D. The use of pit bulls was not justified even though Charlie was burglarizing Rodney's home.

B. Rodney could use the pit bulls because he has a right to protect his possessions. Answer B is the best answer because it is incorrect as a defendant may not use lethal force to protect property. Answer A is wrong because pit bulls cannot be used to protect property alone just like a spring. Thus, it is a correct statement. Answer C is wrong because Rodney could have used deadly force if he was home because he would be not only protecting his possessions, but himself. Thus, it is a correct statement. Answer D is wrong because lethal force is not permitted in response to any wrongdoing, but instead is allowed only when human life is at risk. Thus, it is a correct statement.

Joannie and Sally regularly borrow each other's clothes at school. After gym class, Joannie sees Sally's sweater lying on the bench. Thinking that she can borrow it, Joannie puts on the sweater and leaves to go to class. The gym teacher sees this and has Joannie charged with larceny. What is Joannie's best defense? A. She did not carry away the sweater. B. She believed she had Sally's consent to wear the sweater. C. She could always return the sweater if she were caught. D. Joannie has no defense.

B. She believed she had Sally's consent to wear the sweater. Answer B is the best answer because Joannie can assert a defense of consent. Answer A is wrong because Joannie did carry away the sweater, even if only a short distance. Pursuant to common law, the slightest movement was sufficient. Pursuant to the Model Penal Code, no movement is necessary if the defendant has exercised unlawful control. Answer C is wrong because if Joannie had stolen the sweater, merely being willing to return it is not a defense to larceny. Answer D is wrong because Joannie arguably has a defense of consent.

Barbara admits that she is a nymphomaniac. A nymphomaniac is a person obsessed with having sex. The police arrest Barbara for solicitation for prostitution. Can she be prosecuted for this offense? A. Yes, because the Supreme Court has never held that the status of being a nymphomaniac is a protected status. B. Yes, because Barbara is being prosecuted for her conduct, not her status as a nymphomaniac. C. No, because Barbara has a constitutional right to engage in sexual conduct. D. No, because Barbara cannot be prosecuted for her status of being a nymphomaniac.

B. Yes, because Barbara is being prosecuted for her conduct, not her status as a nymphomaniac. Answer B is the best answer because Barbara is being charged with prostitution as opposed to having sex. Like Powell v. Texas, Barbara is being prosecuted for her conduct as opposed to her condition. Answer A is wrong because although the Supreme Court has never specifically address the status of a nymphomaniac, it did indicate in Robinson v. California that status alone could not be criminalized. Answer C is wrong because Barbara is not just having sex, but seeking money for sex. Answer D is wrong because Barbara is not being prosecuted just based on her status.

Eric and Carol have been getting on each other's nerves at work. Carol decides she needs to liven things up by playing a practical joke on Eric. When Eric arrives at work, Carol pretends to throw a baseball at his head. However, unbeknownst to Eric, the ball is attached to a string and rebounds to Carol before it hits Eric. Nonetheless, Eric is extremely startled and upset by the incident. Is Carol guilty of assault? A. Yes, because Carol acted recklessly. B. Yes, because Carol intended to and did create a reasonable apprehension in Eric that he would be hit by the ball. C. No, because Carol never intended to hit Eric with the ball. D. No, because Eric suffered no physical injury.

B. Yes, because Carol intended to and did create a reasonable apprehension in Eric that he would be hit by the ball. Answer B is the best answer because if Carol did intend to create a reasonable apprehension in Eric that he would be hit by the ball, then she is guilty of assault by placing Eric in apprehension of an imminent battery. Answer A is wrong because even though Carol may have acted recklessly, she needed to intend to cause reasonable apprehension in the mind of the victim. Answer C is wrong because a defendant can be guilty of assault even if she never intends to physically harm the victim. An intent to cause reasonable apprehension of imminent physical harm is sufficient. Answer D is wrong because it does not matter for an assault charge whether the victim is actually injured. If the victim is injured, the defendant can be charged with battery.

Jane and Sydney both hate Paula. While Paula sleeps, Jane stabs her and Sydney shoots her. Both Jane and Sydney are charged with murdering Paula. Under the Model Penal Code A. neither is guilty because there can be only one cause of a criminal harm. B. both are guilty because they were the actual and legal cause of Paula's death. C. only Jane is guilty because she acted first against Paula. D. only Sydney is guilty because her acts broke the chain of causation for Jane.

B. both are guilty because they were the actual and legal cause of Paula's death. Answer B is the best answer because both Jane and Sydney are the proximate cause (or legal cause) of Paula's death. Answer A is wrong because pursuant to the Model Penal Code there can be multiple, concurrent causes of criminal harm. Answer C is wrong because the test is not simply who acted first. Although Sydney did not shoot Paula until after Jane had stabbed her, Sydney still acted with the culpability required for murder. Answer D is wrong because Jane is not excused from criminal culpability just because Sydney is also culpable. There was nothing about Paula's death that was so remote or accidental that it would have a just bearing on Jane's culpability.

Puya is an activist against the war. He feels that every day the war threatens to kill more innocent victims. In an effort to try to stop the war, Puya decides to organize a group of protestors to break into the local ROTC office and destroy the equipment inside. When he is apprehended, Puya argues that he was faced with a choice of evils. He could either allow the killing of innocent people or destroy the ROTC office in protest. Puya's necessity defense is likely to A. succeed if the government is fighting an unjust war. B. fail because he could have protested the war by writing to his Congressional representative. C. succeed if he chose the lesser of the two evils. D. succeed because the alternatives might not be as effective as what he did.

B. fail because he could have protested the war by writing to his Congressional representative. Answer B is the best answer because if Puya disagrees with the war, he should write his congressional representative. Answer A is wrong because disagreement with government policy does not grant one a license to violate the law. Answer C is wrong because even if Puya is right and the war is a greater evil, he should be seeking change through the normal democratic processes. Answer D is wrong because a defendant is required to choose a lawful alternative over an unlawful alternative, even if the defendant does not believe it would be as effective as breaking the law. Unless it is so ineffective that it is really no alternative at all, the legal option comes first.

Annette is swimming at the beach. While she is swimming, she is attacked by Frankie. The lifeguard sees the assault, but does not come to Annette's aid. Annette drowns. When charged with her murder Frankie claims that the lifeguard's failure to do his duty relieves Frankie of responsibility for the drowning. Frankie's argument will most likely A. fail because there is no duty to rescue others under U.S. law. B. fail because the lifeguard's inaction did not constitute a superseding, intervening act. C. succeed because the lifeguard should have saved Annette. D. succeed because Frankie was not the proximate cause of Annette's death.

B. fail because the lifeguard's inaction did not constitute a superseding, intervening act. Answer B is the best answer because the lifeguard's duty to save Annette did not break the chain of causation. In other word, the lifeguard's failure to rescue her did not create a superseding, intervening duty. That does not mean that the lifeguard could not be criminally charged for failing to rescue Annette. He could be charged in addition to Frankie. Answer A is wrong because it inaccurately state American law. As a general rule, there is no duty to rescue, but there are exceptions for those who have duties. For example, parents have a duty to rescue children if feasible. Police officers have a duty to rescue the public if feasible. Lifeguards have a duty, based on contract, to rescue drowning swimmers at their jobs. Answer C is wrong because the lifeguard's duty to save Annette did not break the chain of causation. The lifeguard's failure to rescue her did not create a superseding, intervening duty. Answer D is wrong because Frankie was indeed both the actual cause and proximate cause of Annette's death. Because Frankie's actions led directly to Annette's death, there was no intervening act that would shield him from criminal culpability.

Hicks has heard that Rowe has set out to kill his old enemy, Colvard. When Hicks sees Rowe approach Colvard, Hicks yells to Colvard, "Take off your hat and die like a man." As Colvard starts to take off his hat, Rowe shoots him dead. Rowe is then shot by the police and Hicks is prosecuted for Colvard's murder as an accomplice. Hicks is most likely A. guilty because his words may have had the effect of encouraging Rowe to shoot Colvard. B. guilty if he intended his words to encourage Rowe to shoot Colvard. C. guilty if Rowe would not have shot Colvard unless Hicks had yelled his words. D. not guilty because mere words are not enough to aid and abet a murder.

B. guilty if he intended his words to encourage Rowe to shoot Colvard. Answer B is the best answer because words alone may be enough if they are intended as encouragement of the criminal act and if they are capable of providing assistance. Thus, Hicks can be guilty if he intended for his words to promote the homicide. Answer A is wrong because the focus should be on what Hicks intended as opposed to the effect of his words. That effect may be one clue as to his intent, but there must be proof of his purpose. Answer C is wrong because it is not necessary that the accomplice's actions or words be the sole or primary cause of the accomplice's actions. Any efforts that are capable of helping are sufficient. Answer D is wrong because this is a legally inaccurate statement as mere words are enough to aid.

Sonny and his girlfriend, Cher, illegally sell prescription painkillers. Unbeknownst to Sonny, Cher gets the painkillers by forging a physician's signature on prescription pads and filling the different prescriptions at different pharmacies. When Sonny and Cher are apprehended, Sonny is charged with conspiracy to sell drugs, forgery, and the illegal purchase of prescription drugs. Sonny contests the charges. Sonny is A. guilty of conspiracy and the unlawful purchase of prescription drugs. B. guilty of conspiracy, the unlawful purchase of prescription drugs, and forgery. C. guilty as an accomplice to all of Cher's crimes. D. not guilty of any of the crimes because Cher did not tell him about the forgeries.

B. guilty of conspiracy, the unlawful purchase of prescription drugs, and forgery. Answer B is the best answer because regardless of whether a conspirator knows of his co-conspirator's crimes, the conspirator is guilty if the crimes were during the course of and in furtherance of the conspiracy. Here, Sonny and Cher would not have been able to acquire the drugs to engage in their illegal sales unless Cher had obtained the drugs. It is reasonably foreseeable and a natural consequence of the conspiracy that Cher would use any means, legal or illegal, to obtain the painkillers. Answer A is wrong because it does not hold Sonny responsible for the forgeries. Pursuant to the Pinkerton rule, Sonny is responsible for the crime committed by his co-conspirator. Answer C is wrong because it provides the wrong rationale. Specifically, Sonny is not responsible because he is an accomplice. He does not have the actus reus or mens rea for accomplice liability. Answer D is wrong because regardless of whether a conspirator knows of his co-conspirator's crimes, the conspirator is guilty if the crimes were during the course of and in furtherance of the conspiracy.

The Roaring Stones play a concert in a crowded nightclub. Many of the attendees realize that anything can happen in a crowded club, but they shove their way in anyway. To please the crowd, the nightclub owner decides to use some indoor fireworks to punctuate the band's performance. The fireworks misfire and cause the curtains in the club to burst into flames. As the customers rush to flee the burning club, several are trampled to death. The nightclub owner is charged with involuntary manslaughter. The nightclub owner is guilty of involuntary manslaughter if A. he was on notice of the risk of using indoor fireworks, but disregarded those risks during the show because he was willing to take any risk to bring in more patrons. B. he should have realized the risk caused by his extremely negligent behavior, especially when there was no good reason for him to take the risk. C. he acted in the heat of passion. D. he premeditated the deaths of the patrons. E. the victims did not contribute to the negligent behavior.

B. he should have realized the risk caused by his extremely negligent behavior, especially when there was no good reason for him to take the risk. Answer B is the best answer because the owner should have realized the risk by his use of fireworks, which is sufficient to support a charge and conviction for involuntary manslaughter. Answer A is wrong because if the defendant was actually on notice and subjectively disregarded the risks of his behavior, he will likely be guilty of murder as opposed to manslaughter. Answer C is wrong because a heat of passion killing, although a type of manslaughter, is not the type of killing ordinarily classified as involuntary manslaughter. Instead, heat of passion killings are typically voluntary manslaughter or manslaughter. Answer D is wrong because a premeditated killing is first-degree murder not manslaughter. Answer E is wrong because unlike tort law, criminal law does not recognize any contributory negligence of the deceased as a defense to homicide.

Judy and Donna go swimming. Judy is aware that Donna is not a great swimmer. Nonetheless, she takes Donna to a beach with strong currents. She then pushes Donna into the water. Her goal is to frighten Donna a little and show off her own swimming skills. Because of the strong currents, Donna starts to drown. However, before she actually does, Donna is rescued by a lifeguard. Judy is charged with attempted murder. Under the majority approach, Judy is A. guilty of attempted murder because she was aware that Donna might be overcome by the currents. B. not guilty of attempted murder because her goal was to frighten, not kill, Donna. C. guilty of attempted murder because she acted with malice when she pushed Donna into the water. D. not guilty of attempted murder because a reasonable person would realize that a lifeguard would probably save Judy.

B. not guilty of attempted murder because her goal was to frighten, not kill, Donna. Answer B is the best answer because pursuant to common law, a defendant must have the purpose (specific intent) to accomplish the crime. Here, Judy does not have the purpose to kill Donna, and thus cannot be guilty of attempted murder. *Pursuant to the Model Penal Code knowledge of the likely harmful result is sufficient.* Answer A is wrong because attempt, in this case for murder, requires more than Judy's awareness that Donna might drown. It must be established that Judy purpose in pushing Donna into the water was to drown her. Here, Judy's purpose was simply to scare Donna as opposed to kill her. The mens rea for murder is lower than the mens rea for attempted murder. Answer C is wrong because it assumes that the mens rea standard for attempted murder is the same as for a completed murder. The mens rea for murder is lower than the mens rea for attempted murder. Answer D is wrong because it focuses on the reasonable person's mens rea, not the intent of the defendant. The purposefulness standard is not an objective standard. It is a subjective standard that requires prosecutors to prove that the defendant acted purposefully to cause harm.

Malone is apprehended outside the sports stadium selling basketball tickets. Malone believes that it is illegal to scalp tickets, but he is willing to take the risk because he can make so much money reselling tickets. An officer charges him with attempted scalping of tickets. In researching the law, Malone's lawyer discovers that the jurisdiction repealed its scalping laws ten years ago. He argues that Malone is not guilty of attempted scalping because it was impossible for him to commit that crime. Under the common law, Malone should A. succeed because he did not intend to violate the law. B. succeed because it was legally impossible to scalp tickets. C. fail because he tried to violate the law. D. fail because he is guilty of attempted scalping.

B. succeed because it was legally impossible to scalp tickets. Answer B is the best answer because this scenario is a situation of true legal impossibility. No matter what he does, Malone cannot violate a law that does not exist. Answer A is wrong because Malone did intend to violate the law. This response is factually inaccurate and should rule this option out. Be careful of responses that are inconsistent with the facts provided. Answer C is wrong because impossibility always applies when the defendant thinks he may be engaging in conduct that will violate the law. Nonetheless, in the legal impossibility cases, a defendant's mistaken belief is not enough even if the tries to break a law that does not exist. Answer D is wrong because Malone cannot be guilty of attempting anything that is not a crime. Perhaps he is guilty of trespassing or some other minor offense, but he is not guilty of attempted scalping.

Elizabeth is being prosecuted for the attempted murder of Alexander. At Elizabeth's trial which of the following facts would be most likely to lead a jury to render a verdict of not guilty? A: Alexander was already dead when Elizabeth shot him, although Elizabeth believed him to be alive. B: Alexander was alive when Elizabeth shot him, although Elizabeth believed that he was already dead. C: Elizabeth's gun was unloaded when she aimed it at Alexander and pulled the trigger, although Elizabeth believed it to be loaded. D: Intending to poison Alexander, Elizabeth put a harmless substance into his drink, although Elizabeth believed that the substance was lethal.

B: Alexander was alive when Elizabeth shot him, although Elizabeth believed that he was already dead. Rationale: Answer B is the best answer because a person is guilty of a criminal intent when, with the specific intent to bring about a criminally prohibited result, he or she comes substantially close to achieving that result. Thus, all attempts are specific intent crimes. This means that, although murder may be committed without the intent to kill, attempted murder may not. If the defendant believed that the victim was already dead, she could not have intended to kill him and so could not be guilty of attempted murder Answer A is incorrect because the defendant's intent to kill the victim could make him guilty of attempted murder even though the fact that the victim was already dead made murder factually impossible. Answer C is incorrect because the defendant's belief that the gun was loaded could establish that he had the specific intent to kill the victim, even though the fact that the gun was unloaded made it factually impossible for him to accomplish the result that he intended. Answer D is incorrect because the defendant's belief that the substance was a poison could help establish that he had the specific intent to kill the victim, even though the fact that the substance was harmless made it impossible for him to accomplish the intended result.

Frank broke into a mechanics shop to steal some tools. Because it was dark, he used a lighter to see where things were inside the shop. The lighter's flame ignited gasoline fumes from an engine in the shop causing afire that burned down the shop. In a common law jurisdiction, the district attorney charged Frank with maliciously burning the mechanics shop with the intent to deprive the mechanic of the shops assets. In addition to the above facts, what must the prosecutor establish, if anything, to convict Frank of burning the mechanic's shop? A: Frank's purpose in igniting the lighter was to burn down the shop. B: Frank knew he could set the mechanics on fire, but ignored the risk and ignite the lighter inside the shop. C: Frank carelessly burned down the mechanics shop. D: Frank always hated the mechanic and therefore wanted to burn the shop down, so he ignited the lighter.

B: Frank knew he could set the mechanics on fire, but ignored the risk and ignite the lighter inside the shop. Rationale: Answer B is the best answer because the minimum intent required for most crimes is the standard of recklessly or maliciously. In those crimes, the defendant must actually consider the risk that the property would be burned and disregard that risk.Here, Answer A provides that scenario. See Regina v. Faulkner, 13 Cox. Crim. Cas. 550 (1887). By contrast, when a defendant acts in an unthinking manner, that defendant may have acted negligently or carelessly instead of recklessly or maliciously. Answer A is wrong because the standard of maliciously does not require this high level of intent. Answer C is wrong because the legal standard of maliciously requires that a defendant act more than just carelessly. Answer D is wrong because mens rea refers to a defendant's intent, not motive. Maliciously, does not require that Frank have an evil motive to burn down the mechanics shop. In other words,his motive is irrelevant, if he acted with the requisite intent.

Consider the following offenses: I. Assault. II. Attempted murder. III. Battery. IV. Manslaughter. Which of these offenses can be charged when a murder charge is inapplicable because of the year and a day rule? A: I and II only. B: I, II, and III only. C: I, III, and IV only. D: I, II, III, and IV.

B: I, II, and III only. Rationale: Answer B is the best as the defendant cannot be charged with a homicide, including manslaughter, but assault, attempted murder, and battery are all offenses which a defendant can be charged with in that situation. See Russell L. Weaver, et al., Criminal Law A Contemporary Approach 160 (2d ed. 2014). Answer A is incorrect because it does not include battery. See Russell L. Weaver, et al., Criminal Law A Contemporary Approach 160 (2d ed. 2014). Answers C and D are incorrect because they include manslaughter, which is a homicide and thus impermissible. See Russell L. Weaver, et al., Criminal Law A Contemporary Approach 160 (2d ed. 2014).

Jason met a girl in a club where they were dancing and drinking. Jason was too inebriated to drive, so the girldrove him home. They had consensual sexual intercourse. In their state, it was illegal to have sex with a femaleunder the age of 18 years. The girl was 16 years old. Jason thought that the girl was at least 18 years old. Whichof the following facts would be most important to the outcome if the state prosecutes Jason for statutory rape? A: The girl told him she was 18 years old, misrepresenting her age. B: Jason was lucid enough at the time to know that he was having sex with the girl. C: The club has a policy allowing only people 18-year-old and older to enter. D: A reasonable sober person would have thought the girl was at least 18 years old.

B: Jason was lucid enough at the time to know that he was having sex with the girl. Rationale: Answer B is the best answer because statutory rape is generally a strict liability offense and thus the prosecution does not have to establish that a defendant had any specific or general intent. Even if the jurisdiction viewed the offense as a general intent offense requiring only the intent to have sexual intercourse, Jason still would have been guilty. Regardless of the jurisdiction's approach, Jason is ultimately guilty because he engaged in sexual intercourse with a female who was too young to consent provided that he knew that he was engaging in sexual intercourse. Answer A is incorrect because it is irrelevant whether the girl told Jason that she was over 18 years old as this offense is a strict liability offense. Answer C is incorrect because it is irrelevant whether Jason knew the girls age as this offense is a strict liability offense. Answer D is incorrect because it is irrelevant whether Jason knew the girls age even if a reasonable person would not have known it, and even if she told him that she was over 18 years old as this offense is a strict liability offense.

Rachel wanted to put out a hit on her archenemy. After talking with the right people, she got in touch with Zack. Rachel offered to pay Zack $25,000 to kill her archenemy. Unfortunately for Rachel, Zack was an undercover cop who promptly arrested her. Rachel was charged with conspiracy to commit first degree murder. In a Model Penal Code jurisdiction, how is the jury most likely to rule? 7 A: The jury will find Rachel guilty because she had the intent of killing her husband. B: The jury will find Rachel guilty because she offered to pay Zack to kill her archenemy. C: The jury will find Rachel not guilty because neither Rachel nor Zack had taken a substantial step toward killing the archenemy. D: The jury will find Rachel not guilty because there was no actual agreement.

B: The jury will find Rachel guilty because she offered to pay Zack to kill her archenemy. Rationale: Answer B is the best answer because a conspiracy exists when the defendant or one of the defendant's co-conspirators must engage an overt act in furtherance of the conspiracy to commit the offense; and the defendant must enter into an agreement to commit the offense, which need not be explicit but may be tacit. At common law, the conspiracy could not exist if one of the co-conspirators was pretending to agree to the conspiracy. The Model Penal Code does not have that restriction and, thus Rachel can be convicted of a conspiracy even though Zack is an undercover cop who never intended to agree to the conspiracy. It should be noted that Rachel would be guilty of solicitation too for seeking someone to kill her archenemy. Answer A is incorrect because Rachel's intention alone to have her archenemy killed is insufficient basis for a conviction for conspiracy. Answer C is incorrect because conspiracy does not require that a substantial step be taken for the conspiracy to have occurred. Instead, for the prosecution to establish a conspiracy the defendant or one of the defendant's co-conspirators must engage an overt act in furtherance of the conspiracy to commit the offense; and the defendant must enter into an agreement to commit the offense, which need not be explicit but may be tacit. Answer D is incorrect because the Model Penal Code allows for a conviction for conspiracy even if one of the parties is pretending to agree to enter the conspiracy.

Two drivers, Lisa and Richard were in a car accident when Richard collided his car into Lisas car. At the time of the accident, Richard was drunk and driving negligently. Richard was injured and rendered unconscious. Lisa became anxious and fled the accident scene without checking on Richard, providing him with any first aid, or contacting the police. Eventually, the police discovered Lisas involvement. In this jurisdiction, there is a statute criminalizing fleeing the scene of an accident. In most jurisdictions, could Lisa be successfully prosecuted for failing to do anything to assist Richard? A: Yes, because Lisa had an ethical obligation to assist Richard after injuring him. B: Yes, because most jurisdictions have statutes criminalizing fleeing the scene of an accident. C: No, because Lisa had no duty to assist Richard as he was the negligent cause of the accident. D: No, because there is no general duty to assist accident victims.

B: Yes, because most jurisdictions have statutes criminalizing fleeing the scene of an accident. Rationale: Answer B is the best answer because most jurisdiction have law criminalizing fleeing the scene of an accident in part as such persons have a legal duty to provide reasonable aid as well as contact law enforcement about the accident. Answer A is incorrect because, as a legal mater, there is no ethical or moral obligation to provide assistance. Answer C is incorrect because laws criminalizing fleeing the scene of an accident create a legal duty to stop and provide assistance or contact law enforcement regardless of any fault for the accident. Answer D is incorrect because laws criminalizing fleeing the scene of an accident constitute an exception to the general principle that a person does not have a duty to assist accident victims.

Sonya was a cat burglar specializing in art work. She approached her friend Phoebe about joining her in the burglary of a work of Picasso as a lookout, but Phoebe declined because she said that she did not want to go to jail. Sonya later proposed that she would provide Phoebe with 20% of the proceeds of the sale of the Picasso on the black market in exchange for a place to hide after the theft. Afterwards, Sonya hid at Phoebe's home for weeks before selling the Picasso on the black market. Eventually both Sonya and Phoebe were arrested. Phoebe was charged with conspiracy to commit burglary. Is Phoebe guilty of conspiracy? A: Yes, because she was an accessory to the burglary. B: Yes, because she agreed to give Sonya a place to hide in exchange for 20% of the sale of the Picasso. C: No, because she refused to participate in the actual commission of the burglary. D: No, because her agreement to allow Sonya to stay at her home after the burglary was not per se unlawful.

B: Yes, because she agreed to give Sonya a place to hide in exchange for 20% of the sale of the Picasso. Rationale: Answer B is the best answer because conspiracy is an agreement by two or more persons to commit a crime. Here, Phoebe has an interest in the criminal enterprise, such as a percentage of the sale of the stolen property based on her agreement to provide Sonya with a place to hide may have engaged is a conspiracy. Answer A is incorrect because a conspiracy to commit a burglary is complete when a conspirator agrees with another person to commit the burglary. In other words, the conspiracy is a separate offense from the burglary itself. Here, the fact that a Phoebe is guilty of burglary is not relevant to the question of whether she engaged in a conspiracy to commit a burglary. Answer C is incorrect because even though Phoebe refused to engage in the burglary, she had an interest in the criminal enterprise, such as a percentage of the sale of the stolen property based on her agreement to provide Sonya with a place to hide may have engaged is a conspiracy. Answer D is incorrect because Phoebe's agreement was per se unlawful insofar as she knew that Sonya would be hiding in his apartment to escape capture for the theft of the Picasso as because Phoebe had an interest in the criminal enterprise based on the percentage of the sale of the stolen property she stood to receive.

A state law bars the sale of firearms to anyone under the age of 18 years old. That state's courts have interpreted the law to be a strict liability offense and vicarious liability for the sale to anyone underage. Monica, a gun store owner, advised all of her employees to check for identification of all sales of firearms. Eric went into Monica's store and asked for a Glock. The sales clerk asked Eric how old he was, and he replied that he was 22. Eric was actually only 17, but looked much older. After the sales clerk sold Eric the gun, the district attorney charged Monica with selling firearms to minors. Should Monica be convicted for selling firearms to minors? A: Yes, even though Monica had instructed her sales clerks not to sell a firearm to a minor. B: Yes, even though the sales clerk did not intend to sell a firearm to a minor. C: No, because the sales clerks inquired about Eric's age. D: No, because Eric lied about his age.

B: Yes, even though the sales clerk did not intend to sell a firearm to a minor. Rationale: Answer B is the best answer because an employer can be vicariously liable for employees who violate a strict liability offense regardless of the employee's intent. Here, the sales clerk's intent is irrelevant because the law barring sales of firearms to minors is a strict liability offense that has also interpreted to include provided vicarious liability. Answer A is incorrect because Monica's diligent instructions do not prevent her from being found vicariously liable for this charge of offense. Answer C is incorrect because a person is culpable pursuant to a strict liability offense even if one takes steps to avoid the criminal act. Here, even though the sales clerk tried to ascertain Eric's age, that is no defense. Consequently, it is no defense for the prosecution of Monica based on vicarious liability. Answer D is incorrect because it does not matter in a strict liability offense if the victim lies as the defendant must not engage in the criminal conduct regardless. Here, the sales clerk is culpable if the sale is made to Eric regardless of her role in facilitating the sale by lying about his age.

Karl and Donald have arranged to deliver cocaine to Laura. Karl drives the car as Donald directs him to the delivery point. As they approach the drop-off point, Karl senses that Laura has arranged to rip off the cocaine and leave Karl and Donald with nothing. Karl backs up quickly to get away. As he does, he accidentally hits Molly, a little old lady who had the misfortune of crossing the road at that moment. Molly dies. Under the common law approach, what crimes is Donald guilty of? A. Murder only. B. Only conspiracy to murder and conspiracy to distribute cocaine. C. Attempted distribution of cocaine, conspiracy to distribute cocaine, and murder. D. Accomplice liability, attempted distribution of cocaine, and conspiracy to distribute cocaine.

C. Attempted distribution of cocaine, conspiracy to distribute cocaine, and murder. Donald conspired with Karl. There need not be an express agreement for form a conspiracy. Here there was concerned action that demonstrates a conspiracy. Donald had the purpose to distribute cocaine and thus a conspiracy to do so. Donald is also guilty of attempted distribution of cocaine as he and Karl have taken a substantial step toward delivering it. Thus, Answer C is the best answer. Answer A is wrong because it does not state that Donald is also guilty of conspiracy. Answer B is wrong because it does not recognize that Donald has accomplice liability for the attempted distribution of cocaine and liability under felony murder for the death of Molly. Answer D is wrong because it holds that Donald is guilty of accomplice liability. Accomplice liability is not a separate crime, but instead is a theory by which a defendant can be found guilty of a substantive crime.

Valerie, Patsy, and Barbara go to a party. Barbara gets a headache and asks for a glass of soda. Unbeknownst to her, someone spikes her soda with a hallucinogen. Patsy takes a bit of the "special brownies" that are being served. Patsy assumes the brownies have a little marijuana in them. What she doesn't know is that they are also laced with LSD. Finally, Valerie tells her host that she does not drink or take drugs because she is afraid of the potential effect. The host pokes her in the rear end with a little needle that he tells her is a "pick-me-up." It turns out to be heroin. Minutes later, Valerie, Patsy, and Barbara are found setting fire to guests' cars in the front of the house. Which of the defendant(s) could successfully claim involuntary intoxication? A. Only Barbara, because she did not know her soda had been spiked. B. Patsy and Barbara, because they did not anticipate they might ingest a drug. C. Barbara and Valerie. D. None of them because there is always a risk of becoming intoxicated at a party.

C. Barbara and Valerie. Answer C is the best answer because Barbara has a good argument for an involuntary intoxication defense as she was unaware of ingesting a hallucinogen. Similarly, Valerie has a good argument for an involuntary intoxication defense as she was forcibly injected with heroin. On the other hand, Patsy expected the brownies to have some narcotic even though she did not expect them to have LSD. In other words, she had the intent to violate the law against using illegal narcotics Answer A is wrong because if fails to recognize that Valerie had a valid argument for an involuntary intoxication defense. Answer B is wrong because Patsy cannot assert an involuntary intoxication defense when she thought she was ingesting just marijuana instead of LSD. In other words, she had the intent to violate the law against using illegal narcotics Answer D is wrong because it is too broad. The act of going to a party by itself does not prevent the use of an involuntary intoxication defense. The use of words like never or always in an answer typically will be a trigger that the answer is incorrect.

Brian owns a pitbull named "Killer." Brian received Killer as a gift from his parents. He has no idea why his parents named the dog "Killer," but Brian thinks it is a funny name for such a wonderful, gentle pet. Killer has always been very tame with Brian. At most, he barked at the postal carrier, but Killer is usually very well tempered. One day, Brian takes Killer to his local playground and lets him run loose among the small children. When one of the children, little Sammy, decides to pet Killer, the dog suddenly turns on Sammy and tears him to shreds. Brian is charged with murder. A. Brian is guilty of murder because he should have known Killer was a dangerous dog. B. Brian is guilty of murder because he knew his dog was likely to kill. C. Brian is guilty of murder if he realized that Killer was a dangerous dog. D. Brian is guilty of murder because he is strictly liable for the acts of his dog. E. Brian is not guilty because he did not command the dog to kill the child.

C. Brian is guilty of murder if he realized that Killer was a dangerous dog. Answer C is the best answer because in order to have malice, Brian needed to realize that the risk that his dog would seriously hurt or kill someone. If he realizes that risk, he has malice and is guilty of murder. If he does not realize that risk, then he is not guilty. Answer A is wrong because the standard for malice is not that the defendant "should have known" that his dog was dangerous, but that "he did know and took the risk." The former language is a negligence standard. Answer B is wrong because, although it applies the correct standard, it is inaccurate factually. There is no evidence that Brian knew his dog would kill the boy. Answer D is wrong because there is no strict liability for homicide. Answer E is wrong because, if Brian knew that the dog posed a danger, it would not matter whether he ordered the dog to attack or not. Brian is not guilty of murder because he did not realize that Killer was a dangerous dog.

Darnell agrees to sell his car to Jamie. Darnell tells Jamie that his car has only 60,000 miles on it. In fact, Darnell has rolled back the odometer on the car from 140,000 miles. Jamie pays Darnell $12,000 for the car. The car falls apart shortly thereafter. What crime, if any, has Darnell committed? A. Embezzlement. B. Larceny by trick. C. False pretenses. D. Highway robbery.

C. False pretenses. Answer C is the best answer because false pretences applies when the defendant obtains title to the property through deceit. In other words, if the victim thinks that she is giving something to the defendant forever, the crime involved is false pretenses. Here, Jamie thinks that she is giving Darnell the $12,000 forever. Thus, he has committed false pretenses. Answer A is wrong because embezzlement only applies when the defendant misappropriates property already in his possession. Jamie's money only came into his possession after he gave her the car. Answer B is wrong because if the defendant only gets possession because the victim does not know he is given up something forever, larceny by trick is the applicable crime. Here, Jamie knows she is giving the money permanently in exchange for the car. In other words, Darnell obtains "title" to the money. Answer D is wrong because Darnell's actions are not robbery as he has not used force or the threat of force to obtain Jamie's property.

Katie is desperate for money. Her son, whom she adores, wants to go to camp with his friends. However, Katie does not have the money to send him. She decides to get the money she needs by buying insurance on an old boat her family has and surreptitiously burning it to collect the insurance proceeds. Believing her son is at school, Katie goes down to the empty lot where the boat is kept. She looks around to make sure no one is nearby and then lights the boat on fire. After the boat is destroyed, she is horrified to discover that her son skipped school that day and was hiding out in the boat with his friends. The friends survived, but her son was killed during the fire. Under the felony-murder rule, Katie is A. Guilty of murder because she acted with callous disregard for human life. B. Guilty of murder because she should have realized her son could have been hiding in the boat. C. Guilty of murder because her son died during her commission of a felony. D. Guilty of involuntary manslaughter because a reasonable person would have realized that there is always a risk of harm when one sets a fire. E. Not guilty because she never intended to harm another person.

C. Guilty of murder because her son died during her commission of a felony. Answer C is the best answer Katie killed her son during the commission of the felony of arson. Answer A is wrong because the felony murder rule does not require proof of malice. Answer B is wrong because the felony murder rule does not even require that the defendant should have been aware of the risk. The defendant's responsibility for the murder is automatic, regardless of whether another person would have been aware of the risk to human life. Answer D is wrong because the felony murder rule does not require the prosecution to prove any level of mens rea for the homicide and because it provides the standard for involuntary manslaughter when one is using the mens rea approach to levels of homicide. Answer E is wrong because the felony murder rule does not require that the defendant intend to harm anyone to be guilty of murder.

John won't give up. For the last three months, he has asked Miriam to date him. She has steadfastly refused. He demands that she come out of her house and discuss the issue with him in private. When she agrees, he gently leads her out of the house to the porch. As they are standing there, a lightening bolt comes out of the blue and strikes Miriam dead. John is charged with murder. Is John legally responsible for Miriam's death? A. Yes, because she would not have been standing on the porch but for John's demand that she do so. B. Yes, because he had no right to demand that Miriam date him. C. No, because John's actions were not the legal cause of Miriam's death. D. No, because Miriam should be responsible for the acts that led to her death.

C. No, because John's actions were not the legal cause of Miriam's death. Answer C is the best answer because John's actions were not a direct enough cause of Miriam's death to hold him legally responsible. Answer A is wrong because "but for" causation addresses only half of the issue regarding causation. There must still be legal or proximate cause. Answer B is wrong because regardless of whether John had a right to demand that Miriam date him, there is a question of whether he has really done anything to cause her death. Answer D is wrong because criminal law generally does not hold the victim accountable for harm suffered by the victim based on a defendant's conduct. Unlike tort law, criminal law does not have the concept of contributory negligence.

Warner and his buddy, Trent, have decided to rob the bank. Warner is very nervous about the caper, so Trent suggests that they drink a few beers before they get started. By the time they rob the bank, nothing is bothering Warner. Boosted by his liquid courage, Warner robs the bank. Is Warner entitled to an intoxication defense? A. Yes, because he might not have robbed the bank if he had been sober. B. Yes, because Trent was the one who suggested that they drink before the robbery. C. No, because he had already formed the intent to rob before he drank. D. No, because voluntary intoxication is never a defense.

C. No, because he had already formed the intent to rob before he drank. Answer C is the best answer because if Warner had already formed the intent to rob, merely blunting his senses by drinking alcohol would be insufficient to trigger an intoxication defense. Answer A is wrong because the mere speculation that Warner would not have otherwise robbed the bank does not create an intoxication defense. Answer B is wrong because Warner is an adult who could have refused Trent's offer of alcohol. By agreeing to drink, especially as part of his plan to build up his courage to rob the bank, Warner cannot assert an intoxication defense. Answer D is wrong because voluntary intoxication can be a defense, but here it is not as Warner drank alcohol after he had already formed the intent to rob the bank. The use of words like never or always in an answer typically will be a trigger that the answer is incorrect.

Wylie works as a gasoline station attendant. There has been a rash of late afternoon bank robberies in the area. At 4:30 P.M., a car with no license plates and a driver dressed suspiciously pulls into the station. Although Wylie is fairly sure that the driver is the serial bank robber, he fills up the car with gas. The driver then proceeds to rob Security Pathetic Bank. Is Wylie guilty of bank robbery? A. No, because he was not present at the time that Security Pathetic Bank was robbed. B. Yes, because he profited by the bank robber's purchase of gasoline. C. No, because his purpose in selling gas was not to facilitate robbery. D. Yes, because he suspected the car driver was a serial bank robber.

C. No, because his purpose in selling gas was not to facilitate robbery. Answer C is the best answer because there is insufficient evidence that Wylie had the purpose to help in a bank robbery, even if he did sell gas to the robber. Charging him with bank robbery would cast the net of accomplice liability too broadly given all the ways in which people may inadvertently assist in a crime. Answer A is wrong because a person can be an accomplice even though that person is not present at the scene of the crime. Answer B is wrong because it assumes that everyone who profits from a crime is automatically an accomplice. It may be profiting from a crime may be some evidence of the defendant's interest in the criminal activity, here, the small amount of profit from the gas sale does not establish that Wylie had an interest in the robbery. Answer D is wrong because it states the wrong mens rea standard for accomplice liability. Suspicion is not enough. Wylie must have the purpose to promote or facilitate the bank robbery.

Robert and Douglas went riding on Robert's motor boat. Robert said that there was a great area by a rocky outcrop to enjoy water skiing. Douglas, concerned, pointed out the signs near the outcrop which read, "DANGER - Do not swim, boat, or water ski." Robert insisted and drove the boat toward the rocks. After drinking several beers they had onboard, Douglas decided to try waterskiing. Because Douglas had become drunk, he improperly attached the water skis, lost control, and drowned. Is Robert guilty of involuntary manslaughter? A. Yes, because Robert insisted on illegally boating in a dangerous location. B. Yes, because Robert is responsible for all results of his illegal act. C. No, because the cause of Douglas' drowning was his being drunk. D. No, because Douglas was contributorily negligent in assessing the risk.

C. No, because the cause of Douglas' drowning was his being drunk. Answer C is the best answer because there must be a causal relationship between the illegal act and the death for an involuntary manslaughter conviction. Douglas did not drown as a result of boating or waterskiing in a location where those activities were illegal. Douglas fell into the water and drowned as a result of his intoxication. If Douglas had been boating in another area, he would still have drowned, so his proximity to the rocky outcroppings was unrelated to his drowning. Thus, Robert's actions are not a "but for" cause of the drowning. Answer A is wrong because violating the rule against boating or waterskiing near the rocks was not the cause of Douglas' death. Robert would be responsible if the drowning had occurred while Douglas skied and Robert, while operating the boat, steered in a manner that caused Douglas to crash into a rocky outcropping. In that case, Robert would be liable because he was in control of the boat and had insisted on boating and skiing in an unsafe location. Answer B is wrong because Douglas is responsible for his own actions, including drinking excessively while on the lake. Robert would be responsible if the drowning had occurred while Douglas skied and Robert, while operating the boat, steered in a manner that caused Douglas to crash into a rocky outcropping. In that case, Robert would be liable because he was in control of the boat and had insisted on boating and skiing in an unsafe location. Answer D is wrong because the tort defense of contributory negligence does not exist in the criminal law.

Austin Powers intends to steal trade secrets from his competitor at a trade conference. He intentionally buys a briefcase that looks identical to his competitor's so that he can swap briefcases during their meeting and obtain valuable information. However, Austin accidentally picks up his own briefcase and leave the meeting. He is charged with attempted theft of the trade secrets. Which of the following is correct? A. Powers has a mistake of fact defense because he mistakenly picked up the wrong briefcase. B. Powers has a pure legal impossibility defense because it is not illegal to take one's own property. C. Powers has no impossibility defense under the Model Penal Code because if the circumstances were as he believed them to be, he would have taken his competitor's briefcase. D. Powers is not guilty of attempt because it was factually impossible for him to steal his own briefcase.

C. Powers has no impossibility defense under the Model Penal Code because if the circumstances were as he believed them to be, he would have taken his competitor's briefcase. Answer C is the best answer because it properly states the Model Penal Code standard for deciding whether there has been an attempt. If the facts were as Powers believed them to be, then he grabbed someone else's briefcase and committed theft. Consequently, he is guilty of attempted theft and does not have an impossibility defense. Answer A is wrong because this is not a mistake of fact situation. Mistake of fact would apply if the defendant thought he was taking his own property, but mistakenly took another's property. Here, Powers intends to take another's property and thus has sufficient mens rea for the crime. Consequently, mistake of fact does not apply. Answer B is wrong because pure legal impossibility would only apply if it were not a crime to take another's property. Because this type of theft is a crime, legal impossibility is inapplicable here. Answer D is wrong because factual impossibility is not a defense for an attempt offense pursuant to either the common law or the Model Penal Code.

Rudy's life has been a nightmare. He has never had a successful social life because of a large, physical deformity on his face. All his life he ahs been taunted by people calling him names like "Elephant Man" or "Frankenstein." Some have even attacked him. Even after psychological therapy, he rarely has been able to leave his home because the constant ridicule tends to set him off into an uncontrollable rage. One day, Rudy reluctantly ventures out to buy some food at the market. While he is there, a group of people start to point at him and laugh. Scare and furious, Rudy explodes and kills an innocent bystander. Under the Model Penal Code A. Rudy has a full defense to murder because he was provided by the group's laughter. B. Rudy is entitled to a manslaughter instruction because there was legally adequate provocation. C. Rudy is entitled to a manslaughter instruction because he suffers from an extreme emotional disturbance. D. Rudy is guilty of murder.

C. Rudy is entitled to a manslaughter instruction because he suffers from an extreme emotional disturbance. Answer C is the best answer because based on Rudy's personal history, he would be entitled to this instruction. The focus is on whether Rudy had an extreme mental or emotional disturbance for which there is a reasonable explanation or excuse that negates a murder charge. It does not matter that there was not legally sufficient provocation or that he killed an innocent bystander instead of one of those who taunted him. Answer A is wrong because Rudy does not have a full defense. Regardless of whether it is pursuant to the Model Penal Code or common law, provocation is never a full defense. It only mitigates and reduces the defendant's crime from murder to manslaughter. Answer B is wrong because merely mocking a person, or pointing at him is not generally a legally adequate provocation. A reasonable person is not expected to be so enraged as to kill another human just because of embarrassment. Answer D is wrong because the focus is on whether Rudy had an extreme mental or emotional disturbance for which there is a reasonable explanation or excuse that negates a murder charge. Here, there is based on Rudy's personal history. It does not matter that there was not legally sufficient provocation or that he killed an innocent bystander instead of one of those who taunted him.

Baker is driving his new sports car. While he is driving, the accelerator sticks and his car begins to speed. Suddenly, he sees blue lights behind him. The only thing that Baker can do to stop is to turn off the engine. When he does, the police officer promptly comes over and gives him a ticket for speeding. Baker offers the following defenses to the charge. Which one is most likely to succeed? A. The car is new and Baker is unfamiliar with its features. B. Baker did not intend to violate the speeding laws. C. The car had an unexpected and uncontrollable mechanical failure. D. Baker was going the same speed as other drivers.

C. The car had an unexpected and uncontrollable mechanical failure. Answer C is the best answer because even though a strict liability crime does not require a culpable mens rea, it still requires a voluntary actus reus. If the accelerator's problem was truly unanticipated and uncontrollable, Baker may be able to argue that the did not commit a voluntary act that led him to speeding. Instead, he would argue that the accelerator is a mechanical device that literally moved on its own. If the finder of fact found Baker credible, the lack of any voluntary actus reus would be legitimate defense to a strict liability offense. Answer A is wrong because speeding is a strict liability offense and thus there is no requirement that the defendant intend or anticipate the wrong that occurred. Answer B is wrong because speeding is a strict liability offense and thus Baker's intent is irrelevant. It does not matter whether he wanted he wanted to be the safest or slowest driver on the road. If he voluntarily did the criminal act, i.e. sped, he is guilty. Answer D is wrong because it is generally not a defense to a crime that other people are committing the same offense. If everyone is speeding, then they are all guilty of the crime unless they are acting involuntarily.

Alexander, Polly, and Quentin have conspired to rob a bank. As they are driving to the robbery, Polly gets cold feet. She tells Alexander and Quentin to drop her off on their way to the bank because she changed her mind about the robbery. After they drop her off, Alexander and Quentin proceed to rob the bank. However, they are foiled in their attempt when the police happen upon the robbery and stop it. Alexander, Polly, and Quentin are charged with conspiracy to rob the bank and attempted robbery. Polly claims she withdrew from the conspiracy so she is not guilty of any crimes. Assuming Polly voluntarily and completely withdrew from the conspiracy, which of the following is true? A. Under the Model Penal Code, Polly is not guilty of any crime because she withdrew from the conspiracy. B. Under the common law approach, Polly is not guilty of any crime because she withdrew from the conspiracy. C. Under the Model Penal Code, Polly is only guilty of conspiracy, but not guilty of the attempted robbery. D. Under both the Model Penal Code and common law, Polly is guilty of conspiracy and attempted robbery.

C. Under the Model Penal Code, Polly is only guilty of conspiracy, but not guilty of the attempted robbery. Answer C is the best answer because by withdrawing from the conspiracy, Polly could limit her co-conspirator liability for the attempted robbery, but she is still guilty of the conspiracy because she did not attempt to stop the bank robbery. It was coincidental that the police thwarted it. Answer A is wrong because even assuming that Polly voluntarily and completely withdrew from the conspiracy, she would still be guilty of the conspiracy to rob the bank because she did nothing to thwart the conspiracy. All she did was drop out of it. At a minimum, Polly is still guilty of conspiracy even pursuant to the Model Penal Code. Answer B is wrong because the common was stricter in its standard. Even if a defendant withdrew from a conspiracy, she would still be guilty of the original crime of the conspiracy. Answer D is wrong because Polly did enough to withdraw from the conspiracy and terminate co-conspirator liability. However, she did not do enough to escape liability altogether. See Laurie Levenson, Glannon Guide to Criminal Law 302-03 (4th ed. 2015).

Defendant Weiss is charged with kidnapping. He and his buddies decided to help law enforcement by personally apprehending a person they believed committed a murder. It turns out that Weiss was wrong is his assessment and he and his friends were charged with kidnapping. Kidnapping is defined in that jurisdiction as "knowingly confining a victim without authority of the law." Weiss claims that he honestly believed that he was acting within the authority of the law when he apprehended the wrong suspect. Prosecutors argue that Weiss's mistake of law is no defense. Does Weiss have a mistake of law defense? A. No, because mistake of law is no defense. B. No, because mistake of law is only a defense if a defendant really was authorized to seize the victim. C. Yes, because Weiss did not know that he was without legal authority to seize the victim. D. Yes, because Weiss had a good motive in seizing the victim.

C. Yes, because Weiss did not know that he was without legal authority to seize the victim. Answer C is the best answer because the terms of the statute requires that Weiss know that he is not authorized to seize the victim. Because he does not know this, he is not guilty of the crime. This situation is similar to mistake of fact except the "fact" that Weiss does not know is what the law does and does not authorize. Answer A is wrong because it only states the general rule and does not consider the exceptions to that rule. Answer B is wrong because if Weiss actually was authorized to seize the victim, he would not be guilty of violating the statute and would not need to raise a mistake of law defense. In other words, logically, this answer makes no sense. Answer D is wrong because even defendant's with good motives can still break the law. The issue is not his motive, but whether Weiss had the requisite intent for the crime.

Donald is hiking in the mountain when he learns that a storm is headed his way. Donald has time to hike down the mountain and avoid a dangerous situation. However, Donald does not choose that option. Instead, he breaks into a cabin to shelter himself from the coming storm. When he is charged with trespass, Donald tries to argue a necessity defense. Donald's necessity defense is likely to A. fail because the threat of physical harm was not imminent. B. fail because he had a lawful alternative. C. fail because Donald was not under an imminent threat and he had a lawful alternative. D. succeed because Donald was faced with a choice of evils.

C. fail because Donald was not under an imminent threat and he had a lawful alternative. Answer C is the best answer because it establishes that Donald had a lawful alternative of walking down the hall as well as the threat to him was not imminent. Answer A is wrong because Donald's necessity defense would not succeed as he also had a lawful alternative of walking down the hill. Answer B is wrong because Donald's necessity defense would not succeed as the harm that he faced was not imminent. Answer D is wrong because it wrongly assumes that whenever a defendant is faced with a choice of evils, the defendant automatically gets to argue necessity.

Molly owns a pharmaceutical manufacturing plan. She tries to maintain the highest standards of production and cleanliness in her operation. Each of her employees goes through very detailed training on how to operate the machines that prepare the pharmaceuticals. The facility is routinely inspected for cleanliness. Molly personally tells her employees that their number one priority should be the safe production of pharmaceuticals. One night, when Molly is home asleep, a rat sneaks into the operating plant and finds its way into a vat of pharmaceuticals being produced. Unfortunately, an inspector finds the rat the next day doing the backstroke in the vat and charges Molly with a misdemeanor offense that holds an owner of pharmaceutical manufacturing facility strictly liable for any contamination. Molly is fined $10,000 for the strict liability offense of operating a contaminated facility. If Molly claims that she was unaware of the rat and did everything she could to ensure her facility was not contaminated, her defense will A. succeed because Molly generally operated a clean plant. B. succeed because only the night watchman who could have spotted the rat should be responsible. C. fail because Molly is automatically responsible for the violation. D. fail because all crimes involving pharmaceutical plants are strict liability offenses.

C. fail because Molly is automatically responsible for the violation. Answer C is the best answer because Molly is charged with a strict liability crime that imposes vicarious liability on the owners. Answer A is wrong because it is no defense that Molly generally did not violate the law. Although that may be a factor at sentencing, it does not relieve Molly of criminal responsibility for this violation. Answer B is wrong because although she was not physically at the plant when the vat was contaminated, Molly is the legal operator of the facility and thus vicariously liable for her employees' errors. Answer D is wrong because it is too absolute and overly broad of a statement. There is nothing to indicate whether or not all crimes involving pharmaceutical plants are strict liability crimes.

Leland and Jane have a casual conversation about how great it would be if they had as much money as their boss. Leland tells Jane, "It wouldn't be that hard to share his money. All we would need to do is to take it out of the safe when he goes home for the day." Jane, just nods her head in response. At the end of the work day, Jane sneaks into her boss's office. Inside, she sees Leland emptying the safe. Jane helps him by stuffing some of the money into her clothes and carrying it out of the business. Leland has arranged for Myron to act as a lookout for the theft. When their boss discovers the loss, he checks the film from a video camera hidden above the safe. It shows Leland, Jane, and Myron in his office. They are all charged with conspiring to steal from the boss. Jane claims that she never agreed to participate in a conspiracy. Jane's argument is likely to A. succeed because she never said she would help Leland steal from her boss. B. succeed because she never knew that Myron was in the conspiracy. C. fail because the three of them engaged in concerted action. D. succeed, because a nod of the head is insufficient to show membership in a conspiracy.

C. fail because the three of them engaged in concerted action. Answer C is the best answer because by engaging in concerted action, Jane demonstrated that she was joining the conspiracy. Of course, some act that assists co-conspirators is insufficient without meeting the conspiracy mens rea requirement. Answer A is wrong because it is not required that a conspirator expressly agree orally or in writing to be a member of a conspiracy. A co-conspirator can show in many different ways, including concerted action, that she has joined a conspiracy. Answer B is wrong because it is not required that the defendant know all of the members of the conspiracy. Not all conspirators must join at the same time. A defendant who joins an ongoing criminal conspiracy is automatically responsible for the actions of everyone in the conspiracy who acts in furtherance of that conspiracy. Answer D is wrong because the nod of a head can be enough of an actus reus for a conspiracy.

Sally was in a terrible marriage. Her husband, Marc, constantly beat and humiliated her. In front of the children, he would threaten to kill her and them. Over the course of their ten-year marriage, Marc had broken Sally's arm, ribs, and fractured her skull. Yet, she could not escape him. Even when she went to a battered women's shelter, he would find her. One horrible day, Marc came home from work and told Sally, "You've got 20 minutes to live, you slut." He then grabbed a beer and sat down in his favorite chair, with a shotgun by his side. As he was drinking his beer, Marc fell asleep. While he was sleeping, Sally quickly grabbed the gun and shot Marc in the head. She was charged with murder. If Sally claims self-defense, she is likely to A. succeed under the traditional approach to self-defense. B. succeed because Marc deserved to die. C. fail under the traditional approach to self-defense, but may succeed under a subjective approach of the immanency requirement. D. fail under any standard because she could have called the police while her husband slept.

C. fail under the traditional approach to self-defense, but may succeed under a subjective approach of the immanency requirement. Answer C is the best answer because pursuant to the subjective standard, the jury would be directed to consider Sally's entire situation and past in determining whether, from her perspective, the threat from Marc was imminent. Because the threat was omnipresent and she had never been able to escape him, she might be successful arguing the subjective standard. Answer A is wrong because the traditional approach would require that the threat to Sally be objectively imminent. With Marc asleep, although she feared him, it was not reasonable to believe that he would kill her at that moment. Answer B is wrong because self-defense is not to be used to reap one's own justice and the imminency requirement is a check on vigilante justice. Answer D is wrong because Sally would have a chance for a valid assertion of self-defense pursuant to the subjective standard.

Margaret realizes that the dam at the top of the hill is about to break. If it breaks one way, it will kill many people in the town. However, if Margaret dynamites the dam to break in another direction, it will flood Perry's farmlands and destroy all his crops, but no people. Margaret dynamites the dam, knowing full well that Perry's crops will be destroyed. She wishes she had had time to talk to Perry before taking action, but the dam was going to break at any minute. If Margaret is charged with destroying Perry's property, she A. has no defense because she did not have Perry's consent to flood his lands. B. has no defense because the threat was not imminent. C. has a defense because she saved the lives of the townspeople. D. has a defense because Perry is unlikely to complain.

C. has a defense because she saved the lives of the townspeople. Answer C is the best answer because Margaret is allowed the necessity defense as she chose between the lesser of two evils. Answer A is wrong because while normally a crime to destroy property without the owner's permission, the whole point of a necessity defense is to allow a defense when the defendant is forced to choose, like here, between the lesser of two evils. Answer B is wrong because the facts dictate that the dam could break at any moment. Thus, there was no imminent threat. Answer D is wrong because if Margaret is not entitled to argue necessity, then she has committed a crime regardless of whether the victim chooses to prosecute.

Consider the following statements regarding complicity. I. At common law, principals received the death penalty if convicted. II. At common law, accessories received the death penalty if convicted. III. At common law, acquittal of the principal barred prosecution of an accessory in a felony case. IV. At common law, acquittal of the principal barred prosecution of an accessory in a misdemeanor case. Which of the statements above is correct? 1 A: I and II. B: I and III. C: I, II, and III. D: All of the above.

C: I, II, and III. Rationale: Answer C is the best answer because notwithstanding the acquittal of the principal a co-defendant charged as an accessory could be prosecuted for an underlying misdemeanor case. Consequently, all of the other answers are incorrect. See generally Russell L. Weaver, et al., Criminal Law A Contemporary Approach 197-98 (2d ed. 2014).

Consider the following elements: I. The defendant either must intend to agree to commit the offense or must intend to accomplish the commission of the offense's goal. II. The defendant or one of the defendant's co-conspirators must engage an overt act in furtherance of the conspiracy to commit the offense. III. The defendant must enter into an agreement to commit the offense, which need not be explicit but may be tacit. Which of these elements are necessary to establish a criminal conspiracy? 1 A: I and II only. B: I and III only. C: II and III only. D: All of the above.

C: II and III only. Rationale: Option I describes an erroneous interpretation of the mens rea for conspiracy. As a matter of law, the prosecution must establish both the mental in order to obtain a conviction for conspiracy. In other words, the defendant must intend to agree as well as must intend to accomplish the commission of the offense's goal. Option II describes the overt act requirement that is typically applied in most jurisdiction. This overt act does not have to be a criminal act itself. For example, a co-conspirator could engage in an over act by leaving a getaway car parked in front of the bank with the keys in it. By itself, this act may not be a criminal act, but could nonetheless be an overt act in furtherance of a conspiracy to rob a bank. Option III describes the fundamental actus reus of conspiracy, which is the act of agreement does not need to be explicit but may be implicit. Consequently, Answer C is the best answer because it incorporates Option II and III and excludes Option I.

Justine was driving to the mall when she lost consciousness as well as control of her car. She struck Armando, a pedestrian in the crosswalk, who subsequently died. It was subsequently learned that, at the time of the accident, Justine had lost consciousness because she had food poisoning. Prior to the accident Justine did not realize that she had food poisoning or that it could cause her to lose consciousness. Should the prosecutor charge Justine with vehicular homicide for Armando's death? A: Yes, because Justine could still be convicted even though there was no voluntary act. B: Yes, because Justine committed a voluntary act when she chose to drive to the mall. C: No, because Justine did not act voluntarily in killing Armando. D: No, because Justine was driving her car in a safe and reasonable manner prior to the accident.

C: No, because Justine did not act voluntarily in killing Armando. Answer C is the best answer because, as a matter of law, an individual can only be punished criminally if one acted consciously and voluntarily. See Martin v. State, 17 So.2d 427 (Ala. 1944); see also People v. Decina, 138 N.E.2d 799 (N.Y. App. Ct. 1956). Here, Justine cannot be convicted as the accident was caused by her food poisoning and resulting loss of consciousness, which was an involuntary act. See Fulcher v. State, 633 P.2d 142 (Wyo. 1989). Answer A is incorrect because, as a matter of law, an individual can only be punished criminally if one acted consciously and voluntarily. See Martin v. State, 17 So.2d 427 (Ala. 1944); see also People v. Decina, 138 N.E.2d 799 (N.Y. App. Ct. 1956). Therefore, Justine cannot be punished because she had no voluntary act designed to render herself unconscious. See Fulcher v. State, 633 P.2d 142 (Wyo. 1989). Answer B is incorrect because although Justines decision to drive was a voluntary act that clearly precipitated the accident and pedestrians death, it was the food poisoning and resulting loss of consciousness that caused the accident. There is no evidence that the food poisoning and resulting loss of consciousness was a voluntary act. See Fulcher v. State, 633 P.2d 142 (Wyo. 1989). Answer D is incorrect because this is the standard concerning negligence from tort law and does not address whether there was any actus reus for purposes of a criminal prosecution.

Melvin is an epileptic who has not had a seizure for over years and diligently takes his medication. One day,while he was driving his car, he had a seizure and struck another car killing a driver. The district attorney charged Melvin with reckless homicide. Melvin argues to the jury that he did not realize that he would have a seizure given that he has not suffered one for over 10 years. If the jury finds Melvin credible, would he be convicted of reckless homicide? A: Yes, because Melvin should have considered the risks to others on the road when he drove with a seizure disorder. B: Yes, because an ordinary person would have realized that driving with a seizure disorder could lead to a deadly accident. C: No, because Melvin never realized that he might harm the other driver. D: No, because Melvin did not have the purpose to kill the other driver.

C: No, because Melvin never realized that he might harm the other driver. Rationale: Answer C is the best answer because if the jury finds that Melvin did not consider the risk to the other driver, then Melvin has not acted recklessly. Consequently, Melvin has not satisfied the mens rea requirement for the charged crime. Answer B is wrong because it describes the mens rea standard of negligently. The crime and the question, however, require that Melvin act recklessly in order to be convicted. Answer A is wrong it states Melvin should have considered the risk, which describes the mens rea standard of negligently. Reckless requires that Melvin considered the risk and ignored it. Answer D is wrong because it sets as tandard that is too high. The crime does not require that Melvin acted purposefully, but merely recklessly. If Melvin did act purposefully then he would have also acted recklessly. However, Melvin argued to the contrary in this case.

Which of the following is not a culpable mental state at common law based on our casebook reading? A: Feloniously. B: Maliciously. C: Recklessly. D: Willfully

C: Recklessly. Answer C is the best answer because recklessly is a term from the Model Penal Code. See Russell L. Weaver, et al. Criminal Law A Contemporary Approach 80-81 (2d ed. 2014). All of the rest were terms from common law. The common law culpable mental states are maliciously, corruptly, negligently, and willfully. See Russell L. Weaver, et al., Criminal Law A Contemporary Approach 77 (2d ed. 2014).

Police discovered an envelope containing child pornography in Bill's dorm room. A friend had put it in Bill's room 3 days earlier. A state statute made it a felony to knowingly possess child pornography. In addition to the above facts, what must the prosecutor establish, if anything, to convict Bill for felony possession of child pornography? A: The prosecutor does not need to establish anything else to obtain a conviction. B: The prosecutor must establish that Bill should have known that the envelope contained child pornography, and that Bill handled the envelope. C: The prosecutor must establish that Bill knew or believed that the envelope contained child pornography. D: The prosecutor must establish that Bill knew or believed that the envelope contained child pornography, and that Bill handled the envelope.

C: The prosecutor must establish that Bill knew or believed that the envelope contained child pornography. Rationale: Answer C is the best answer because the prosecutor wants to convict Bill of knowingly possessing child pornography.In order for an individual to act knowingly, that person must be cognizant that her behavior is the type of behavior that the law criminalizes. Consequently, Bill could not have acted knowingly unless he knew or believed the envelope held child pornography.Answer A is incorrect because the state law at issue mandates that Bill knew or believed that the envelope held child pornography.Answer B is incorrect because the criminalization of the possession of child pornography typically only needs a showing that the defendant had control of the child pornography for a long enough time to have a chance to end the possession. Therefore, the prosecutor would not have to show that Bill handled the envelope. Moreover, Bills failure to know when he should have known that the envelope held child pornography would be negligence, which would be insufficient to establish the scienter of knowingly.Answer D is incorrect because the criminalization of the possession of child pornography typically only needs a showing that the defendant had control of the child pornography for a long enough time to have a chance to end the possession. Consequently, the prosecutor would not have to show that Bill handled the envelope.

Marybelle has seen her neighbor wearing the most beautiful diamond ring. Marybelle decides that she must have it. As she walks past her neighbor's home one evening, Marybelle sees the ring on the inside of her neighbor's windowsill. Seeing that nobody is home, Marybelle tries to open the window. It is already open a crack and Marybelle is able to open it enough to reach her hand inside. However, the ring is just out of her reach. Marybelle runs home and grabs some tongs, then uses them to reach inside the window. She grabs the ring and runs. However, she is later arrested when her neighbor sees Marybelle wearing her ring around town. Which of the following crime(s) can Marybelle be charged with? A. Larceny. B. Burglary. C. Robbery. D. A and B. E. All of the above.

D. A and B. Answer D is the best answer Marybelle took the personal property of anther with the intent to deprive that person permanently of the property. Thus, she is guilty of larceny in taking the ring. She could also be charged with burglary as that incident happened at night, it was a residence, and at the time she reached inside, she had intent to commit a felony, i.e., to steal the ring. Thus, Marybelle can be charged with both larceny and burglary. Answer A and B wrong because although Marybelle could be charged with larceny, she can also be charged with burglary. She took the personal property of anther with the intent to deprive that person permanently of the property. Thus, she is guilty of larceny in taking the ring. She could also be charged with burglary as that incident happened at night, it was a residence, and at the time she reached inside, she had intent to commit a felony, i.e., to steal the ring. Thus, Marybelle can be charged with both larceny and burglary. Answer C and E are wrong because Marybelle's actions are not robbery as she did not used force or the threat of force to obtain the ring.

Wilma and Bernie have been married for 50 years. Tragically, Wilma is dying a painful death from an incurable disease. Every day, she asks Bernie to help her die so that she can be relieved of her misery. Bernie can't bear to see her suffer. He finally takes matters into his own hands and gives Wilma an overdose of sedatives. Wilma dies peacefully in her sleep. A. Bernie is not guilty of murder because he killed Wilma out of mercy. B. Bernie is not guilty of murder because he killed Wilma at her request. C. Bernie is guilty of murder, but not first-degree premediated murder. D. Bernie is guilty of first-degree murder.

D. Bernie is guilty of first-degree murder. Answer D is the best answer because mercy killings are still first-degree murders. Bernie's motive is irrelevant to whether he had malice here. Answer A is wrong because even if Bernie's motive was to act out of mercy, he still acted with premeditation. Thus, he is guilty of first-degree murder. Answer B is wrong because Wilma's request can actually be used by the prosecution to help prove premeditation and explain Bernie's decision to kill her. Thus, he is guilty of first-degree murder. Answer C is wrong because the facts establish that Bernie killed Wilma in a premediated manner. His motive is irrelevant to whether he had malice here.

Elie returns home after a hard day's work. He catches his wife, Debi, in bed with the gardener. Elie starts laughing and says to the gardener, "if you want her, you can have her. I'd rather mow the lawn." Elie then heads outside. While he is moving the lawn, he reconsiders what he said and decides this would be a good opportunity to get rid of his wife and her lover. He heads back into the house, loads his gun, and walks to the bedroom. Coolly, he takes aim and fires. He kills the lover instantly. If Elie claims he killed in the heat of passion, his defense will most likely A. Succeed because a reasonable person might be provoked to kill if he finds his wife in bed with another man. B. Succeed if Elie's wife and her lover intentionally provoked Elie. C. Fail because Elie may have been provoked to kill his wife, but not the gardener. D. Fail because Elie was not acting in the heat of passion.

D. Fail because Elie was not acting in the heat of passion. Answer D is the best answer because Elie does not get a heat of passion defense because he did not act in the heat of passion. Instead, after some thought, he decides to kill his wife and her lover. Answer A is wrong because it does not matter whether a reasonable man would have been provoked if the defendant himself was not actually provoked. The first requirement for the heat of passion doctrine is that the defendant must actually be in the heat of passion. Answer B is wrong because the intent of the victims is irrelevant. The first requirement for the heat of passion doctrine is that the defendant must actually be in the heat of passion. Answer C is wrong because it does not accurately reflect the facts here. Elie was not provoked to kill either of them.

Ray and his buddies are cocaine distributors. Ray is the brains behind the operation. He keeps lists of customers, obtains the cocaine from a wholesaler, and divides up the distribution duties to his buddies. Jan delivers the actual cocaine to the customers while Karl acts as a bodyguard. During one of their recent deliveries, Jan and Karl needed a quick way to escape when witnesses called the police. Jan's cousin, Fenton, gave them his car to use to flee. If the old common law approach is applied, which of the following classifications is correct? A. Ray is the principal in the first degree, Jan is the principal in the second degree, Karl is the accessory before the fact, and Fenton is the accessory after the fact. B. Ray is the principal in the first degree, Karl is the principal in the second degree, Jan is the accessory before the fact, and Fenton is the accessory after the fact. C. Jan is the principal in the first degree, Ray is the principal in the second degree, Karl is the accessory after the fact, and Fenton is the accessory before the fact. D. Jan is the principal in the first degree, Ray is the accessory before the fact, Karl is the principal in the second degree, and Fenton is the accessory after the fact.

D. Jan is the principal in the first degree, Ray is the accessory before the fact, Karl is the principal in the second degree, and Fenton is the accessory after the fact. Answer D is the best answer. Jan is the one at the crime scene engaging in the actual delivery of drugs so he is principal in the first degree. Karl is also near the scene when the drugs are involved, but as the bodyguard is not engaged in the delivery so he is principal in the second degree. Although Ray is the mastermind, he is not on the scene, but engaged in planning the deliveries so he is an accessory before the fact. Fenton is only involved after the drugs are delivered trying to help them escape so he is an accessory after the fact.

Manuel asks Franco to help him burn down city hall. Franco thinks Manuel is kidding and answer flippantly, "sure." A police officer, who overhears the conversation, immediately arrests both of them and charges them with conspiracy to commit arson. Is Franco guilty of conspiracy? A. Yes, because he agreed to burn down city hall. B. Yes, because he was on notice that Manuel wanted to burn down city hall. C. No, because it only takes one person to burn down city hall. D. No, because Franco was just kidding when he answer, "sure."

D. No, because Franco was just kidding when he answer, "sure." Answer D is the best answer because if Franco was just kidding when he responded to Manuel, then he did not intend to join any conspiracy to burn down city hall. Answer A is wrong because even if Franco said "sure," and Manuel understood that to be an agreement, if Franco did not have the intent to join the conspiracy or have Manuel burn down city hall, he is not guilty of conspiracy. Answer B is wrong because the standard for conspiracy is not negligence or recklessness or even knowing conduct. In most jurisdiction, the defendant must act purposely. Answer C is wrong because it does not matter if the co-conspirator is needed or does anything to help the conspiracy. A conspiracy charge criminalizes the unlawful agreement itself.

Tom is fascinated with movie stars, especially Shannon Stone. One day, he sees her in the grocery store and starts to stare at her. He follows her up and down the aisles. Tom gives Stone the creeps. She calls the authorities to prosecute Tom for stalking. Stone tells the authorities that even though Tom has never spoken a work to her, he makes her uncomfortable and she fears that the next time they run into each other in the market, he will snap and hurt her. Is Tom guilty of stalking? A. Yes, because he made Stone feel uncomfortable. B. Yes, because he followed Stone in the store. C. No, because Tom has not come dangerously close to harming Stone. D. No, because Tom does not pose a credible threat to Stone.

D. No, because Tom does not pose a credible threat to Stone. Answer D is the best answer because the crime of stalking only applies when there is a credible threat of harm to the victim. Here, Tom's behavior, although making Stone uncomfortable, has not demonstrated any credible threat of harm. Answer A is wrong because merely making someone uncomfortable does not constitute the crime of stalking. The defendant must pose a credible threat and there must be evidence of intent to harass the person. Answer B is wrong because it is not a crime to intentionally follow another person unless there is intent to cause fear in the victim by one's behavior. Here, Tom may have felt that it was flattering for Stone to be followed by a fan. Moreover, there is no evidence that a reasonable person would have felt in fear of Tom's actions. A reasonable person would not have been happy and maybe annoyed, but there is insufficient evidence of a credible threat. Answer C is wrong because it confuses the law of attempt and the law of stalking. At common law, one manner of establishing attempt a defendant come in dangerous proximity of completing the crime. On the other hand, stalking punishes behavior that may not reach the threshold of attempt.

Shellie's television has broken. Fortunately for Shellie, she purchased an extended warranty so that she can call the repairman whenever her set needs service. Shellie calls the repairman, who says he will be out in the afternoon to fix her set. Later, the repairman arrives, but he goes to the wrong house. Shellie's neighbor, Byron, has the repairman fix his television instead. Under traditional common law, is Byron guilty of larceny? A. Yes, because he had not paid for the repair service. B. Yes, because his actions involved personal property. C. No, because he was in mere custody of his television. D. No, because he did not take the property of another.

D. No, because he did not take the property of another. Answer D is the best answer because pursuant to a strict common law definition of property, Bryon did not take the property of another. He possibly could be charged and convicted of some other offense, but not common law larceny. Answer A is wrong because stealing services did not constitute larceny pursuant to common law. Answer B is wrong because although Byron's television constitutes personal property, the crime of larceny requires the trespassory taking of another person's property. Because Byron did not take Shellie's television set, he is not guilty of larceny. Answer C is wrong because Byron does not have mere custody over the television, but instead it is actually his television. Moreover, the issue of mere custody only arises when the defendant is charged with temporarily having someone else's items in his possession.

Bryan decides to take his neighbor's Ferrari for a joyride. He sneaks in the car when his neighbor is not looking and starts it with a spare set of keys kept in the glove compartment. While Bryan is cruising, the police pull him over on a traffic violation. Because the car's registration is not in Bryan's name, the officer arrests Bryan for stealing the car. Is Bryan guilty of larceny? A. Yes, because he took the car without permission. B. Yes, because he intended to permanently deprive his neighbor of the car. C. No, because the car had an extra set of keys in it. D. No, because he was only going for a joyride.

D. No, because he was only going for a joyride. Answer D is the best answer because Bryan was only going for a joyride, he did not intend to permanently deprive his neighbor of his car. Thus, he is not guilty of larceny. Answer A is wrong because even if Bryan did not have permission to take the car, he only intended to borrow it. Answer B is wrong because it is counter to the facts. If it is a joyride, then a defendant simply intends to use the car before eventually returning it. Answer C is wrong because criminal law does not blame the victim. Thus, even if the neighbor negligently left keys in the glover compartment, provided that there was no consent given to Bryan, this negligence is irrelevant.

Obama hears a clicking in his luggage. He believes that it is the travel alarm clock that he packed. Little does he realize that someone has mysteriously substituted a ticking bomb for his alarm clock. When airport security inspects Obama's luggage, they discover the bomb. They charge him with illegally possessing an explosive device. Is Obama guilty of the offense? A. Yes, because Obama had a bomb in his luggage. B. Yes, because Obama owned the luggage with the bomb. C. No, because only the person who put the bomb into the luggage could have possessed it. D. No, because he was unaware that the illegal item was in his possession.

D. No, because he was unaware that the illegal item was in his possession. Answer D is the best answer because Obama is unaware of his "possession" of the bomb. The Model Penal Code requires knowledge by the person charged with some type of illegal possession offense. Answer A is wrong because Obama did not know that he possessed the bomb as someone slipped it into his luggage. Answer B is wrong because who owns the luggage is not relevant. Instead, the person charged with possession must be aware of the contraband. Answer C is wrong because it is too limiting. Someone who had the bomb with knowledge could be guilty of possession.

Consider again the case of Regina v. Cunningham, 2 Q.B. 396 (1957), where the defendant almost asphyxiated this neighbor by ripping a gas meter from the wall and thereby allowing gas to seep into the victim's room. If Cunningham claimed that he didn't realize that breaking a gas meter would cause gas to seep and the jury believed him, would he be guilty of the crime of recklessly asphyxiating his neighbor? A. Yes, because an ordinary person would have realized that breaking a gas meter would allow the gas to seep into his neighbor's apartment. B. Yes, because the defendant should have considered the risks to his neighbor when he broke the gas meter. C. No, because the defendant did not have the purpose to kill his neighbor. D. No, because the defendant never realized that he might harm his neighbor.

D. No, because the defendant never realized that he might harm his neighbor. Answer D is the best answer because if the jury finds that the defendant did not consider the risk to his neighbor, then the defendant has not acted recklessly. Consequently, he has not met the mens rea requirement for the charged crime. Answer A is wrong because it describes the mens rea standard of negligently. The crime and the question, however, require that the defendant act recklessly in order to be convicted. Answer B is wrong it states the defendant "should have" considered the risk, which describes the mens rea standard of negligently. Reckless requires that the defendant considered the risk and ignored it. Answer C is wrong because it sets a standard that is too high. The crime does not require that the defendant acted purposefully, but merely recklessly. If the defendant did act purposefully then he would have also acted recklessly. However, the defendant argued to the contrary.

Barclay is surprised to find that he has been charged with rape. The night before, he met a woman in a bar who asked him for a ride home. When they got to her place, he assumed she wanted him to stay the night. Even though he never asked her if he could stay, he claimed that from the look in her eye he could see that she really wanted to have sex with him. Almost immediately, the woman started crying. However, she never actually said the words "no" or "stop." Barclay honestly believed that she was crying because she was so emotional over her good fortune to be with Barclay. In that jurisdiction, a defendant has a defense to rape if he honestly and reasonably believes the victim has consented to sexual intercourse. Does Barclay have a reasonable mistake of fact defense? A. Yes, because he honestly believed that his victim consented to have sex with him. B. Yes, because the woman assumed the risk by asking him for a ride home. C. No, because rape is a strict liability crime. D. No, if Barclay's mistake is not reasonable.

D. No, if Barclay's mistake is not reasonable. Answer D is the best answer because for Barclay to have a viable defense, his mistake must be reasonable (and honestly believed). Ultimately, a jury must decide whether Barclay's actions meet the two-prongs of "honestly believed" and "reasonable" to meet the mistake of fact defense in this jurisdiction. Answer A is wrong because an honest mistake may be enough for may crimes, but it is not enough here because the jurisdiction requires that the mistake be both honest and reasonable. Answer B is wrong because victims do not assume the risk, nor is there any contributory negligence in criminal law. Be wary of these types of misleading answers that are traps, especially based on tort law. Answer C is wrong because rape is not a strict liability offense, but requires intent. However, statutory rape is a strict liability offense.

LeeAnn really loves dogs. She takes in any stray dog she finds. Currently, she is caring for 12 pathetic pooches. LeeAnn's neighbors have warned her that city ordinances limit the number of dogs per household to three. LeeAnn is not worried, however, because the dog catcher in her city sent her a letter last year congratulating her on being "citizen of the year" for her outstanding work in helping to shelter stray animals. LeeAnn interprets this as the dog catcher's approval of her activities. City officials recently issued LeeAnn a citation for violating the ordinance that prohibits more than three dogs per household. Not worried, LeeAnn marches to court with the letter of commendation she has from the city dog catcher. Does LeeAnn have a mistake of law defense? A. Yes, because LeeAnn relied upon the dog catcher's letter. B. Yes, because LeeAnn reasonably relied upon an official interpretation of the law. C. Yes, because an official interpretation of the law by an administrative official is a valid mistake of law defense. D. No.

D. No. Answer D is the best answer because she does not have an official interpretation of the law by an appropriately high governmental official. Answer A is wrong because she relied on information from a governmental official that was not an official interpretation of the law. Instead, it was simply a letter of commendation. Answer B is wrong because she relied on information from a governmental official that was not an official interpretation of the law. If the letter were issued by a top official as an official statement of the law, then she might reasonably rely on it as a defense. Here, there are no facts to support such reliance. Answer C is wrong because although it is a correct statement of the law, it incorrectly assumes that the letter is an official interpretation of the law.

Ellie, a very petite woman, went to the unofficial office happy hour after work. Although she was not generally a drinker, she frequently attended the nightly happy hour to get to know other people in the firm. During the course of her 45-minute stay, Ellie had one glass of white wine. On her short drive home, a police officer lawfully pulled her over. Suspecting Ellie was drunk, the officer conducted a field sobriety test. Ellie's blood-alcohol level was just above the legal limit. Which of the following arguments might successfully be put forth in her defense to a charge of driving under the influence? A. She did not know she was intoxicated. B. She did not intend to drive while intoxicated. C. Any reasonable person would have acted in the same manner, considering the circumstances. D. None of the above.

D. None of the above. Answer D is the best answer because drunken driving in many states is considered a strict liability offense. In other words, a person who is voluntarily intoxicated and driving with a blood alcohol level above the legal limit is going to be held criminally liable. As a strict liability offense, drunk driving crimes require no showing of a particular mental state. See generally Staples v. United States, 511 U.S. 600 (1994). Keep in mind that strict liability crimes are highly unusual in the criminal law because the question of culpable mental state is central to our society's determinations of appropriate criminal sanctions. Answers A, B, and C are wrong because none of the claims would further Ellie's defense that the act alone can culpability. Each of these responses contends that Ellie did not have the requisite mental state of mind to be convicted of driving while intoxicated. Many such criminal statutes have no state of mind requirement, beyond knowledge of driving after consuming alcohol. Although most criminal laws have a specified or implied mens rea requirement, no additional state of mind is required for a conviction in this case. The mere act of drinking and driving has been determined to be of such significance that the act alone can permit culpability.

Bruno works as a collector for the local mafia. He has been given the assignment to collect on a loan made to Mr. Brando. Unfortunately for Brando, he doesn't have the cash that Bruno wants. As a result, Bruno slams Brando against a wall, gives Brando a few strong punches to the gut, and then walks away while saying, "Next time, you'll remember to pay on time." Brando staggers home. A few days later, he dies of a ruptured spleen caused by Bruno's blows. Is Bruno guilty of murder? A. No, because he never intended to kill Brando, as evidenced by the fact that Bruno told Brando that he would be collecting from him in the future. B. No, because a punch to the stomach is not necessarily a fatal blow. C. No, because the killing was not premeditated. D. Yes, because he acted with malice. E. Yes, because he intended to kill Brando.

D. Yes, because he acted with malice. Answer D is the best answer because Bruno has acted with implied malice even though he did not intend to kill Brando as he still intended to cause serious bodily injury. That intent is sufficient to prove malice and thus murder. Answer A is wrong because, although Bruno did not intend to kill Brando, he can be guilty of murder if he intended to cause serious bodily injury. Here, the facts establish Bruno had this intent. Answer B is wrong because even if a defendant does not intend the blow to be fatal, a defendant can be convicted of murder. Malice includes situations in which the defendant only intends to harm his victim, but the victim dies instead. Answer C is wrong because premeditation is a requisite for first-degree murder but not all murders. A defendant may act with malice even when the defendant does not plan in advance to kill the victim. Answer E is wrong because the facts do not establish that Bruno intended to kill Brando.

Barney, a drunk driver, strikes and severely injures Ellis. When Ellis is taken to the hospital, he is mistakenly left in the corner of an overcrowded emergency room. Tragically, he dies of internal bleeding before doctors can treat him. According to the defense experts, if Ellis had been treated in a more timely manner, he could have been saved. Prosecutors charge Barney with murder. Is Barney the proximate cause of Ellis' death? A. No, because the hospital officials were negligent in not treating Ellis sooner. B. No, because Ellis could have been saved if he had been treated in a timelier manner. C. Yes, because Barney started the chain of causation by striking and injuring Ellis. D. Yes, because the hospital staff's actions did not break the chain of causation.

D. Yes, because the hospital staff's actions did not break the chain of causation. Answer D is the best answer because there were no acts or omissions by the hospital staff that would constitute a superseding cause to limit causation. Answer A is wrong because even if the hospital officials were negligent, mere negligence does not break the chain of causation. Here, Barney remains culpable for the victim's death notwithstanding the medical malpractice. Answer B is wrong because even if proper treatment could have saved the victim's life, Barney is not relieved of responsibility for the harm he caused. Answer C is wrong because it is not just that Barney started the chain of causation that renders him criminally responsible, but because that chain was not broken by the actions or omissions of the hospital staff.

Joshua illegally smuggles aliens across the border. His partner, Maria, gives Joshua the names of aliens who are willing to pay to be brought across the border. They share the fee paid by the aliens. Maria has recently come to Joshua with the names of more potential customers. Joshua checks the weather report to see when the moon will be new because it is easier to smuggle people without any moonlight. Maria and Joshua are arrested before either one of them can do anything else. Are Maria and Joshua guilty of conspiracy? A. No, because they have not actually started to transport the aliens. B. No, because they have not taken a substantial step toward transporting them. C. No, because it was not illegal for Joshua to check the weather reports. D. Yes, because they both agreed to smuggle the aliens and there was a least one overt act in furtherance of that conspiracy.

D. Yes, because they both agreed to smuggle the aliens and there was a least one overt act in furtherance of that conspiracy. Answer D is the best answer because both defendants had all the necessary elements for the crime of conspiracy, including the simple overt act of checking the weather report. Answer A is wrong because it is not required that the conspiracy progress to the point where the conspirators are actually transporting aliens. An overt act can be any step after there has been an agreement to commit a crime. Answer B is wrong because substantial step is the standard for an attempt. For an overt act, any act to further the conspiracy would qualify. Answer C is wrong because it is not required that the overt act be illegal in itself. It can be a totally lawful act and still qualify as an overt act.

Assume that in the Goetz case, Goetz is confronted with four youths wielding knives. At just that moment, subway police come running up from behind Goetz, although he does not see or hear them. They easily outnumber the attackers and could subdue them and escort Goetz to safety. Before Goetz can see the officers, he shoots the attackers. If Goetz is in a jurisdiction that applies the duty to retreat rule, is he entitled to use self-defense? A. No, because the arrival of the police eliminated the need to use lethal force in self-defense. B. No, because Goetz could have retreated with complete safety. C. No, because Goetz had a duty to retreat because he was not in his own home. D. Yes.

D. Yes. Answer D is the best answer because even though there is no explanation of why he had the right to use self-defense. However, it is clear that the denial of self-defense responses were wrong. Goetz is entitled to self-defense if he fears that he is in danger and cannot retreat. Answer A is wrong because the duty to retreat arises only when the defendant is aware that a safe retreat is feasible. Here, it is not clear even whether the officers' arrival ended the need for self-defense. Answer B is wrong because the duty to retreat arises only when the defendant is aware that a safe retreat is feasible. Here, it is unclear whether Goetz could even retreat with complete safety and more important, whether he was aware that he could safely retreat. Answer C is wrong because while it is true that the duty to retreat arises outside one's home, location is not the only factor in deciding whether the duty applies.

Steve is considered a neighborhood menace. He regularly leaves old junk in his front yard, including automobiles teetering on blocks, old refrigerators, and bins of sharp scrap metal. Several neighbors have complied to Steve that his junk could easily hurt children who play on the block. In fact, there have been several near misses when kids narrowly escaped being injured by one of the cars that fell off its blocks. Steve just ignores his neighbor's complaints. One day, tragedy strikes. Betty Sue, a young child who lives next to Steve is killed when one of the cars falls off its blocks and onto her. Steve is charged with Betty Sue's murder. In a Model Penal Code jurisdiction, Steve would be A. not guilty of homicide because the death was an unavoidable tragedy. B. guilty of first-degree murder because Steve knew children could be killed. C. guilty of second-degree murder because Steve acted in a grossly reckless manner. D. guilty of murder.

D. guilty of murder. Answer D is the best answer because Steve's extreme indifference provides a basis for charging and convicting him of murder pursuant to the Model Penal Code. Answer A is wrong because not only was the tragedy avoidable, but careless behavior may demonstrate an extreme indifference to human life and support a murder conviction pursuant to the Model Penal Code. Answer B and C are wrong because the Model Penal Code does not have degrees of murder. If this question concerned a common law jurisdiction, then C would have applied because extreme recklessness is a form of malice.

Bob wanted to rob an armored car when it delivered money to a supermarket. Frank watched the armored car deliver money for a week before providing Bob with times and routines of the delivery. Neal was Bob's lookout on the day of the robbery. After the heist, Leon met Bob and took the money to hide it in a storage facility. At common law, who can be charged as principals? A: Bob should be charged as a principal in the first degree, and Leon should be charged as a principal in the second degree. B: Neal should be charged as a principal in the first degree, and Frank should be charged as a principal in the second degree. C: Neal should be charged as a principal in the first degree, and Leon should be charged as a principal in the second degree D: Bob should be charged as a principal in the first degree, and Neal should be charged as a principal in the second degree.

D: Bob should be charged as a principal in the first degree, and Neal should be charged as a principal in the second degree. Rationale: The best answer is D as Bob is the one at the crime scene engaging in the actual robbery of the armored car so he is principal in the first degree and Neal is also near the scene when the robbery happens, but as the lookout is not engaged in the robbery so he is principal in the second degree. See Standefer v. United States, 447 U.S. 10 (1980). Answer A is incorrect because Leon is an accessory after the fact because he is only involved after the robbery in assisting the hiding of the money. Answer B is incorrect because Neal is near the scene when the robbery happens, but as the lookout is not engaged in the robbery so he is principal in the second degree and Frank is an accessory before the fact as he has developed information to assist in the staging of the robbery. Answer C is incorrect because Neal is near the scene when the robbery happens, but as the lookout is not engaged in the robbery so he is principal in the second degree, and Leon is an accessory after the fact because he is only involved after the robbery in assisting the hiding of the money.

Doug was back at his alma mater to attend a football game and was partying outside the stadium with his friends. One of his friends game him a shot of grain alcohol. Doug became very inebriated and stumbled backwards bumping roughly into a woman walking in the parking lot, causing her to fall and break her wrist. The local prosecutor charged Doug with criminal battery. Which of these statements, if it was the only one true, would provide Doug's defense attorney with the most effective argument A: Doug did not intend to become inebriated by drinking the grain alcohol shot. B: Doug did not know that drinking the grain alcohol shot would cause him to stumble. C: Doug did not intend to make contact with the woman. D: Doug had never been overcome previously by the alcohol in one shot.

D: Doug had never been overcome previously by the alcohol in one shot. Rationale: Answer D is the best answer because it provides the best argument that Doug's intoxication was involuntary. Criminal battery consists of the intentional, reckless, or criminally negligent application of force to the body of another. As a general intent crime, it may be committed without any intent to make contact with the victim. While voluntary intoxication is no defense to a general intent crime, involuntary intoxication typically is. A person has become involuntarily intoxicated when his or her intoxication was the result of an unpredictable and grossly excessive reaction to an intoxicating substance. Consequently, if Doug had never been overcome previously by the alcohol in one shot, his defense attorney could argue that his intoxication was involuntary, which would give him a reasonable defense against criminal battery. Of course, this defense might not be successful because a court could find that although the response was unpredictable, it was not grossly excessive. Answer A is incorrect because a person may become voluntarily inebriated even without the intent to become drunk so long as he or she is aware that the substance that he or she is taking has an intoxicating potential. Because Doug was aware that the grain alcohol had some alcohol in it, his intoxication is voluntary even if he did not intend to become drunk. Answer B is incorrect because if Doug's conduct in drinking the grain alcohol shot was reckless or criminally negligent, he could have the necessary mens rea to be guilty of battery (i.e., general intent), even though he did not specifically know what risk he was creating (i.e., that he would stumble). Answer C is incorrect because battery is a general intent crime, which does not require any intent to make contact with another person.

Wendy and Erica worked with the same mean bully. After Wendy was humiliated by this bully, she was commiserating with Erica. Wendy threatened to get even with the bully. Erica agreed with Wendy telling her that the bully deserved to be punished for how the bully treated everyone. Over the weekend, Wendy donned a ski mask and severely beat up the bully. Wendy was apprehended and charged with battery. Did Erica's comments to Wendy create a basis for criminal culpability by Erica? A: Yes, because Erica intended to encourage Wendy to hurt the bully. B: Yes, because Erica encouraged Wendy to hurt the bully. C: No, because the First Amendment protects Erica's comments. D: No, because Erica did not show the requisite intent for Wendy to attack the bully.

D: No, because Erica did not show the requisite intent for Wendy to attack the bully. Rationale: Answer D is the best answer because it is unlikely that a reasonable person would view Erica's comment at the time it was made as encouraging Wendy to hurt the bully. The fact that the words may have encouraged Wendy does not imbue them with the necessary intent to hold Erica criminally culpable. Answers A and B are incorrect because there is no evidence that Erica intended for Wendy to hurt the bully even if she noted that that the bully deserved to be punished. Erica's comment is legally insufficient to establish criminal culpability as a prosecutor must establish intent clearly. Here, there is no such evidence. Answer C is incorrect because the First Amendment is not necessarily a shield to criminal behavior. For example, the criminal offenses of aiding and abetting or solicitation often involve some words. If a defendant could avoid a conviction simply based on the First Amendment, it would be difficult for prosecutors to ever convict anyone of those offenses.

Jessica is friends with both Alicia and Monica. The three of them regularly go to the mall after school. One day, Alicia and Monica decide that they are going to steal some makeup when they go to the mall. Jessica does not say anything about their plan and goes to the mall with them. When Alicia and Monica are arrested, Jessica is arrested too. Pursuant to the modern approach, can Jessica be charged as an accomplice? A: Yes, because Jessica intended to encourage Alicia and Monica to steal the makeup. B: Yes, because Jessica encouraged Alicia and Monica to steal the makeup. C: No, because the modern approach merged the distinctions between principals and accessories. D: No, because Jessica's mere presence at the crime scene does not establish accomplice liability.

D: No, because Jessica's mere presence at the crime scene does not establish accomplice liability. Rationale: The best answer is D because has committed no act in furtherance of the crime by Alicia and Monica. See Lane v. Texas, 991 S.W.2d 90(Tex. Ct. App. 1999). Answers A and B are incorrect because there is no evidence that Jessica intended for Alicia and Monica to steal the makeup. Jessica's presence at the mall is legally insufficient to establish criminal culpability as a prosecutor must establish intent clearly. Here, there is no such evidence. Answer C is incorrect because while this is a true statement, it does not address the question of whether Jessica can be charged as an accomplice.

Ellen was very intoxicated at a huge party being thrown at her sorority. She happened to answer the door when the police arrived at 3 a.m. It was apparent to Officer Hinojosa that Ellen was very drunk. Officer Hinojosa asked her to step outside the police would raid her sorority house. When Ellen walked over to the street to talk with Officer Hinojosa at the police car, she was charged with public intoxication. Should the district attorney pursue a public intoxication charge against Ellen? A: Yes, because the law does not have an actus reus requirement. B: Yes, because Ellen satisfied the actus reus requirement when she got drunk. C: No, because even if the court infers an actus reus requirement, Ellen did not violate the law. D: No, because an actus reus requirement will be inferred by the courts, and Ellens conduct did not constitute a violation of the law.

D: No, because an actus reus requirement will be inferred by the courts, and Ellens conduct did not constitute a violation of the law. Rationale: Answer D is the best answer because an actus reus requirement will not be implied even when the law does not explicitly indicate such a requirement. See Martin v. State, 17 So.2d 427 (Ala. 1944). Although Ellen was voluntarily intoxicated, her decision to enter the public space while intoxicated was not voluntary. Because Officer Hinojosa lured Ellen out of the sorority with the threat of a police raid, there is a good argument that actus reus did not exist. Answer A in incorrect because courts generally imply the actus reus requirement even when the law has not explicitly established such a requirement. See Martin v. State, 17 So.2d 427 (Ala. 1944). Answers B and C are incorrect because while intoxication is typically not a defense to a criminal charge, it does not appear that Ellen intended to go into the public space. Instead, she did so only based on pressure by Officer Hinojosa. Therefore, a jury could find that Ellens decision to go into the street was not voluntary and that the actus reus requirement has not been established. See Martin v. State, 17 So.2d 427 (Ala. 1944).

John owns a convenience store in a marginal neighborhood in which there are lots of prostitutes. He sells a large amount of condoms to many customers who John knows are prostitutes. Is John guilty of conspiracy to engage in prostitution? 6 A: Yes, because he had the necessary mens rea for conspiracy. B: Yes, because he was reckless in selling condoms to known prostitutes. C: No, because it is not per se illegal to possess or sell condoms D: No, because he lacked the requisite mens rea for conspiracy.

D: No, because he lacked the requisite mens rea for conspiracy. Rationale: Answer D is the best answer because John had no intent to promote or engage in prostitution. Instead, he is just selling condoms to any customer, including some who happen to be prostitutes. Additionally, he did not have any agreement with any of the prostitutes for promote or facilitate prostitution. Answer A is the wrong because the sale of the condoms to known prostitutes is not sufficient to establish a conspiracy. There must be an agreement, which is lacking here. Answer B is wrong because the standard, even in these circumstances, is not mere recklessness. The ordinary standard for conspiracy is purposefulness. Even in jurisdictions that reduce the standard, it does not fall lower than knowingly. Answer C is wrong because even the sale of a lawful product can create a stake in an illegal venture, although it requires more of a showing of purposefulness. However, there is no requirement that a co-conspirator provide something that is illegal in all settings.

Nicholas was a meth dealer who cooked his own meth. He knew that most states criminalized the buying cold medicines containing pseudoephedrine unless the buyer signed a log book and provided a state-issued ID. In one state, Nicholas bought a lot of cold medicine containing pseudoephedrine without doing either. However, in that state, a similar type of law barring this type of purchase had recently been repealed. Can Nicholas be charged with an attempt to acquire pseudoephedrine illegally? A: Yes, because factual impossibility is not a valid defense to a charge of attempt. B: Yes, because legal impossibility is not a valid defense to a charge of attempt. C: No, because the repeal made it factually impossibility for Nicholas to have committed an attempt. D: No, because the repeal made it legal impossibility for Nicholas to have committed an attempt.

D: No, because the repeal made it legal impossibility for Nicholas to have committed an attempt. Rationale: Answer D is the best answer because of the doctrine of legal impossibility. For the prosecution to establish an attempt, it must prove that Nicholas had the specific intent to commit the offense and committed an act beyond simply preparing for the offense. However, if the state legislature has repealed the law, then there is no offense addressing Nicholasconduct that is criminal. Answer A is incorrect because factual impossibility concerns a person who seeks to engage conduct that would be a criminal offense if successful, but there is no chance that the individual will succeed. There is no factual impossibility here even though factual impossibility is not a valid defense to a charge of attempt. Answer B is incorrect because legal impossibility is a valid defense to a charge of attempt. Answer C is incorrect because factual impossibility concerns a person who seeks to engage conduct that would be a criminal offense if successful, but there is no chance that the individual will succeed. Here, Nicholas did succeed in what he sought to do, which was buy pseudo-ephedrine, but that was no longer illegal.

State law defined trespass as knowingly entering a private residence without the legal occupant's permission. The courts have interpreted the trespass statute to include entering the yard surrounding a residence. Ben entered his neighbor's yard during a disagreement and was then prosecuted for trespass. In his defense, Ben's attorney maintained that Ben did not know that a yard constituted a residence.Is Ben likely to prevail with this defense? A: Yes, if the Model Penal Code is applied in the jurisdiction. B: Yes, if the common law is applied in the jurisdiction. C: Yes, if the Model Penal Code or common law is applied in the jurisdiction. D: No.

D: No. Rationale: Answer D is the best answer because Ben knew he was entering something and whether that something constitutes a residence is not a factual, but a legal question. Ben's attorney does not argue that Ben knew the legal definition of word residence and mistakenly applied that definition to the yard. His claim is most accurately characterized as ignorance of the law instead of mistake of law. Answer A is incorrect because the Model Penal Code does provide a limited claim of reasonable reliance on official statements made by certain kinds of state actors, but Ben's defense attorney does not make this argument. Answer B is incorrect because the common law did not recognize a claim for ignorance of law. Answer C is incorrect because neither Answer A nor Answer B is correct.

Dave and Jim were drinking at a bar when they decided to leave for another bar. Dave was legally intoxicated, but he drove both of them. Jim did not put on his seatbelt. On the way to the next bar, another driver negligently crashed into Dave's car causing Jim to fly through the front windshield because he was not wearing a seatbelt. After Jim died from his injuries, the prosecutor charged Dave with involuntary manslaughter. How should the court rule on Dave's motion to dismiss the indictment? A: The court should deny the motion because Dave is culpable for all results resulting from his driving while intoxicated. B: The court should deny the motion because Dave was committing the felony of driving while intoxicated. C: The court should grant the motion because Jim was contributorily negligent in failing to use his seatbelt. D: The court should grant the motion because Dave was not the cause of Jim's death.

D: The court should grant the motion because Dave was not the cause of Jim's death. Rationale: Answer D is the best answer because criminal law requires a causal link between the criminal act and the death. Jim did not die because of any criminal act by Dave, but instead because of his failure to use the seat belt. In other words, Dave's driving while intoxicated was not the proximate cause of Jim's death. Answer A is incorrect because Jim is responsible for his own actions, including his failure to use his seat belt. Answer B is incorrect because driving while intoxicated was the not the cause of Jim's death. Instead, the cause was Jim's own failure to use his seat belt. Answer C is incorrect because contributory negligence is a tort law concept that is inapplicable to criminal law

Rachel wanted to put out a hit on her archenemy. After talking with the right people, she got in touch with Zack. Rachel offered to pay Zack $25,000 to kill her archenemy. Unfortunately for Rachel, Zack was an undercover cop who promptly arrested her. Rachel was charged with conspiracy to commit first degree murder. In a common law jurisdiction, how is the jury most likely to rule? A: The jury will find Rachel guilty because she had the intent of killing her husband. B: The jury will find Rachel guilty because she offered to pay Zack to kill her archenemy. C: The jury will find Rachel not guilty because neither Rachel nor Zack had taken a substantial step toward killing the archenemy. D: The jury will find Rachel not guilty because there was no actual agreement.

D: The jury will find Rachel not guilty because there was no actual agreement. Rationale: Answer D is best answer because common law does not allow for a conviction for conspiracy if one of the parties is pretending to agree to enter the conspiracy. Here, Zack is an undercover cop who never intended to enter into the agreement to kill the archenemy. The Model Penal Code does not have this restriction and, thus Rachel could be convicted of a conspiracy even though Zack is an undercover cop who never intended to agree to the conspiracy in such a jurisdiction. Moreover, it should be noted that Rachel could be guilty of solicitation for seeking someone to kill her archenemy. Answer A is incorrect because Rachel's intention alone to have her archenemy killed is insufficient basis for a conviction for conspiracy. A conspiracy exists when the defendant or one of the defendant's co-conspirators must engage an overt act in furtherance of the conspiracy to commit the offense; and the defendant must enter into an agreement to commit the offense, which need not be explicit but may be tacit. Answer B is incorrect because at common law, a conspiracy could not exist if one of the co-conspirators was pretending to agree to the conspiracy. Here, Zack is an undercover cop who never intended to enter into the agreement to kill the archenemy. The Model Penal Code does not have this restriction and, thus Rachel could be convicted of a conspiracy even though Zack is an undercover cop who never intended to agree to the conspiracy in such a jurisdiction. Moreover, it should be noted that Rachel could be guilty of solicitation for seeking someone to kill her archenemy. Answer C is incorrect because conspiracy does not require that a substantial step be taken for the conspiracy to have occurred. Instead, for the prosecution to establish a conspiracy the defendant or one of the defendant's co-conspirators must engage an overt act in furtherance of the conspiracy to commit the offense; and the defendant must enter into an agreement to commit the offense, which need not be explicit but may be tacit.

John, Mike, Sue, and Roger are at a pool party. During the party, a toddler falls into the pool and starts to drown. Everyone sees what is happening, but no one stops to help. John is the toddler's father, Mike is the hired lifeguard, Sue is an off-duty police officer, and Roger is a guest at the party. The prosecution files criminal charges against all four defendants for failing to help the child. Which of the following is correct? A. None of the defendants is guilty because there is no general duty to help another person. B. Only John is guilty because he is the only defendant related to the child. C. Mike and Roger are guilty if they were capable of saving the child without putting themselves at risk. D. All of the defendants are guilty if they were capable of saving the child without putting themselves at risk. E. John and Mike are guilty if they are capable of saving the child without putting themselves at risk.

E. John and Mike are guilty if they are capable of saving the child without putting themselves at risk. Answer E is the best answer because John as the child's father would have a duty and Mike, as the lifeguard, has a contractual duty to help. Roger is a guest and just a bystander. He has no legal duty to assist. Sue is an off-duty police officer. Some jurisdictions would place a duty of off-duty police officers, but there is nothing to indicate that is applicable here. Thus, Sue may have a duty, but it is unclear.

Dundee loves his pet alligator. The alligator is so gentle to him that he never even realized that it could hurt another person. However, Dundee is proved wrong when a neighborhood child wanders into Dundee's yard and becomes the alligator's lunch. Under the Model Penal Code, Dundee should be charged with A. first-degree murder. B. second-degree murder. C. involuntary manslaughter. D. manslaughter. E. negligent homicide.

E. negligent homicide. Answer E is the best answer because pursuant to the Model Penal Code, Dundee has acted negligently as he should have realized the risk to human life, but he did not. Answer A and B are wrong because the Model Penal Code does not have degrees of murder. Moreover, murder, requires that the defendant either "purposely, knowingly or recklessly under circumstances manifesting extreme indifference to the value of human life" cause the death of another. Here, clueless Dundee honestly does not believe that his pet would hurt anyone. Answer C is wrong because the Model Penal Code does not have a separate charge of involuntary manslaughter. Instead, all reckless killings are manslaughter. Answer D is wrong because he has not met the scienter requirement for manslaughter, but negligence. Pursuant to the Model Penal Code, Dundee has acted negligently as he should have realized the risk to human life, but he did not.

Margaret is stopped at the border as she returns from a recent visit to Tijuana, Mexico. Seeing a small jar of white powder on the seat next to her, the police ask Margaret what is in the jar. She tells them that it is some special talcum powder her friends gave her to deal with her terrible skin condition. When the police test the contents in the jar, it turns out to be cocaine. Margaret is charged with "knowingly possessing a controlled substance." If the jury at trial believes Margaret's story, she is A. guilty because she should have known the jar contained cocaine. B. guilty because she knew she was transporting a powder. C. guilty because her mistake was unreasonable. D. guilty because of the deliberate ignorance defense. E. not guilty.

E. not guilty. Answer E is the best answer because if the jury believes that Margaret honestly did not know the powder was cocaine, then she is not guilty of the crime charged. Answer A is wrong because the statute requires that Margaret actually know ("knowingly") as opposed to the lesser standard of should have known ("negligently"). Answer B is wrong because it is insufficient that Margaret knew that she was carrying a powder as the statute requires that she knew ("knowingly possessing a controlled substance") she was carrying cocaine. Answer C is wrong because generally the mistake of fact defense does not require the defendant's mistake to be reasonable. If Margaret honestly does not know that the powder was cocaine even if a reasonable person would know, then she has not violated the statute. However, the more incredulous her position, the less likely the jury will believe her. Answer D is wrong because the facts do not show that Margaret had any suspicions about the powder and deliberately avoided discovering the truth. If there were facts that Margaret had a strong suspicion that the powder was cocaine and carrying it was illegal, but she intentionally avoided confirming such suspicions, then she is presumed to be acting knowingly. However, here the facts do not support this conclusion.

A child falls into a lake and is drowning. There are several adults at the lake: the childs mother; the lakes lifeguard; the childs teacher who happens to be at the lake with her children; and another guest at the lake. All four of these adults see the child drowning, but none of them assist the child, who died. The prosecutor is considering homicide charges. Who should the prosecutor charge? A: The prosecutor should charge only the mother and the other guest. B: The prosecutor should charge only the mother and the lifeguard. C: The prosecutor should charge only the teacher and the lifeguard. D: The prosecutor should charge all of them.

Rationale: Answer B is the best answer because the mother had a duty to her own child, and the lifeguard had a contractual duty to assist people in distress at the lake. See State v. Williquette, 385 N.W.2d 145 (Wis. 1986); see also State v. Williams, 484 P.2d 1167 (Wash. App. 1971); People v. Sealy, 356 N.W.2d 614 (Mich. App. 1984). Answer A is incorrect because although the mother had a duty based on her relationship with the child, but the guest does not have a duty based on her lack of relationship with the child. Answer C is incorrect because although lifeguard had a contractual duty to assist people in distress in the pool, but the teacher does not have a duty based on her relationship with the child as her teacher because they are no in school or a school function. Answer D is incorrect because neither the guest nor the teacher had any duty to assist the child based on their lack of relationship with the child.

James Chow, a 21-year-old college junior, is arrested for driving under the influence on his way home from a weekend fraternity party. James pleads guilty to the offense, but the judge sentences him to the maximum six months in the county jail. At the sentencing hearing, the judge addresses James regarding the sentence: "I'm doing this to teach you a lesson, so that for the rest of your life you'll never get behind the driver's wheel if you've been drinking." Which of the following theories of punishment has the court primarily relied on in sentencing James Chow? A. retribution. B. general deterrence. C. specific deterrence. D. rehabilitation. E. incapacitation.

specific deterrence. Answer C is the best answer because the judge specifically intended to give Chow a lesson. The punishment was designed to ensure that Chow will not commit the same offense ever again. Answer A is wrong because retribution is about punishing past actions as opposed to influencing future behavior. Answer B is wrong because the court's message is to Chow alone as opposed to others who might engage in similar conduct. Answer D is wrong because the punishment is not designed to make Chow a better person. Answer E is wrong because Chow is not being jailed as he is a menace to society.

Defendants have a legal duty to act based on tort law. True or False

False (pp. 55-60)

Maliciously is a Model Penal Code level of culpability. True or False

False It is common law (pp. 77-83)

A conviction for conspiracy to commit a crime does not merge with a conviction for that crime itself for purposes of sentencing. True or False

True (336)

Which of the following is not a way of establishing malice aforethought? A: Extreme emotional disturbance. B: Commission of a battery. C: Extreme indifference to human life. D: Commission of a dangerous felony

A: Extreme emotional disturbance. Rationale: Answer A is the best answer as this was an element of voluntary manslaughter at common law. Answer B is incorrect because the intent to commit a serious bodily injury was a way of establishing malice aforethought. Answer C is incorrect because reckless or extreme indifference, which was known as depraved heart murder, was a way of establishing malice aforethought. Answer D is incorrect because the intent to commit a dangerous felony was a way of establishing malice aforethought.

At common law, which element below was not an element of the offense of assault? A: An attempt to place another person in fear. B: An attempt to commit a battery. C: An intent to cause a battery. D: A present ability to commit a battery.

A: An attempt to place another person in fear. Rationale: Answer A is the best answer because at common law, the element of involving the instilling of fear in another person is one that did not exist at common law for the offense of assault. This element was added in modern criminal codes. See generally Carter v. Commonwealth, 59S.E.2d 28(Va. App. 2004) (en banc); see also Russell L. Weaver, et al., Criminal Law A Contemporary Approach 470 (2d ed. 2014). Answers B, C, and D are all incorrect as they are all elements of common law assault. See People v. Yslas, 27 Cal. 630 (1865); see also Russell L. Weaver, et al., Criminal Law A Contemporary Approach 441-4(2d ed. 2014).

A state law bars the sale of tobacco products to anyone under the age of 18 years old. That state's courts have interpreted the law to be a strict liability offense and vicarious liability for the sale to anyone underage. Billy, a convenience store owner, advised all of his employees to check for identification of all sales of alcohol and tobacco products. Giselle went into Billy's convenience store and asked for a carton of cigarettes. The sales clerk asked Giselle how old she was, and she replied that she was 21. Giselle was only 16, but did look much older. When the sales clerk rang up the carton, Giselle realized that she did not have enough money and left the store. The district attorney charged Billy with an attempt to sell tobacco products to minors. What is Billy's best argument in his defense? A: The sales clerk did not have the mens rea necessary for the offense charged. B: Billy had strictly instructed his sales clerks not to sell tobacco products to minors. C: It was impossible for the sale to have occurred. D: Giselle lied about her age.

A: The sales clerk did not have the mens rea necessary for the offense charged. Rationale: Answer A is the best answer because the sales clerk did not have the necessary mens rea to violate the law because the sales clerk lacked any intent to violate it and thus Billy cannot be vicariously liable for violating the law. A law that is viewed as creating strict liability does not create a strict liability offense for an attempt to violate that law. Instead, the prosecution must demonstrate that the defendant intended to violate the law. Here, the sales clerk did not have any intent to sell to Giselle as a minor, but instead believed she was old enough to buy cigarettes. Consequently, Billy cannot be vicariously liable for an attempt to sell a minor cigarettes if his sales clerk lacked any intent to do so. Answer B is incorrect because Billy's diligent instructions do not prevent him from being found vicariously liable for this charge of attempt. Answer C is incorrect because factual impossibility is not a defense to an offense involving a charge for an attempt. Answer D is incorrect because it does not matter in a strict liability offense if the victim lies as the defendant must not engage in the criminal conduct regardless. Here, the sales clerk is culpable if the sale is made to Giselle regardless of her role in facilitating the sale by lying about her age.

Lester convinced Rick to give him money to buy a firearm to rob a jewelry store. In return for giving Lester this money, Rick would get a third of the jewelry heist. After Rick gave Lester the money for the firearm, Lester had second thoughts. He did not buy a firearm or rob the jewelry store and instead spent the money on a night out with his girlfriend. In a common law jurisdiction, should Lester be convicted of conspiracy to commit robbery? A: Yes, because there was a criminal conspiracy when Rick gave Lester money for the firearm. B: Yes, because there was a criminal conspiracy when Lester asked Rick for the money. C: No, because Lester never purchased a firearm for the robbery. D: No, because Rick did not agree to participate in the robbery of the jewelry store.

A: Yes, because there was a criminal conspiracy when Rick gave Lester money for the firearm. Rationale: Answer A is the best answer because a conspiracy has been established when the co-conspirators reach an agreement to commit a specific criminal act. For the prosecution to establish a conspiracy the defendant or one of the defendant's co-conspirators must engage an overt act in furtherance of the conspiracy to commit the offense; and the defendant must enter into an agreement to commit the offense, which need not be explicit but may be tacit. Here, Lester and Rick have reached an agreement to commit the conspiracy of robbery. Answer B is incorrect because, at common law, Lester's request alone for the money does not satisfy the elements of conspiracy. It would instead be the offense of solicitation. At common law, in order for there to be a conspiracy, Rick would have not only had to been asked for the money, but given it to Lester. Answer C is incorrect because a conspiracy has been established when the co-conspirators reach an agreement to commit a specific criminal act. At common law, it was not necessary for any overt act to be done in furtherance of the conspiracy. Additionally, at common law, the defense of withdrawal is generally unavailable as the conspiracy is established once the agreement is reached. Thus, it is irrelevant that Lester seemingly gave up on the conspiracy by foregoing buying the firearm and instead spending the money on his girlfriend. Answer D is incorrect because a co-conspirator does not have to participate in the criminal conduct for there to be a conspiracy. In other words, Rick did not have to participate in the robbery for there to have been a conspiracy formed. Rick was guilty of conspiracy either he provided the money or when he had an interest in the takings from the robbery. Here, Rick had both.

At the Dollar Tree, Nicholas bought a cheap laser light that had no warning label. Nicholas does not know how the laser works, but is playing with it in his dormitory. He briefly points it at Jacob's eye, causing Jacob to feel a sharp pain and blurred vision. Nicholas is convicted of battery in common law jurisdiction. What is Nicholas' strongest argument in challenging his conviction? A: Nicholas did not cause Jacob any bodily injury or offensive touching. B: Nicholas lacked the mens rea to establish any intent to cause bodily injury. C: Nicholas lacked the mental state to establish criminal negligence. D: All of the above are equally strong arguments in challenging Nicholas'vconviction.

B: Nicholas lacked the mens rea to establish any intent to cause bodily injury. Rationale: Answer B is the best answer because Jacob can argue that he did not have any intent to cause Jacob either bodily injury or to touch offensively because he had no idea that the laser would injure Jacob's eye based on the fact that he did not know how the laser worked. Nicholas' actions, at most, establish a negligent or reckless mental state, but not the intent necessary here at common law. Answer A is incorrect because as a factual matter Nicholas injured Jacob's eye. Therefore, it is unlikely at that an appellate court would dispute that Jacob was not was offensively touched in his eye or that his pain and blurred vision did not constitute a bodily injury. On the other hand, section 210.0(2) of the Model Penal Code defines bodily injury as physical pain, illness, or any impairment of physical condition. See also Adams v. Commonwealth, 53S.E.2d 347 (Va. App. 2000). Answer C is incorrect because, at common law, the prosecution could establish that Nicholas had the criminal negligence mental state. Specifically, a jury could have found his criminal negligence based on his risk-taking behavior in pointing the laser at Jacob's eye. In other words, the jury could presume that Nicholas intended the natural and probable consequences of his action of pointing the laser at Jacob's eye, even if Nicholas did not have that intent. On the other hand, to establish criminal negligence in a Model Penal Code jurisdiction, a prosecutor would have to show that Nicholas should have known about a substantial risk of either offensive touching or bodily injury, and that his actions were significantly beyond how a reasonable person would act. Answer D is incorrect because neither Answer A nor Answer C are strong arguments.

Alan got very high on peyote. As he was walking home, he saw what he thought a bear that was about to attack him so he shot it dead with his gun. In reality, this was not a bear, but a man standing on the side of the road waving. Alan was so stoned that he does not remember anything. A state law establishes that all murders are second degree unless the prosecution can establish premeditation or deliberation, which could then afford the opportunity for the prosecution to charge it as a first degree offense. With the most serious offense listed first following in order of severity to the least serious offense listed last, which offense is the most serious for which a district attorney could prosecute Alan? A: First degree murder. B: Second degree murder. C: Manslaughter. D: Alan cannot be prosecuted for any of these offenses.

B: Second degree murder. Rationale: Answer B is the best answer because Answer C is incorrect because carrying and using a firearm while stoned demonstrates reckless indifferences to a very high risk to human life which meets the legal definition of second degree murder as opposed to the other charges. See State v. Burley, 627 A.2d 98 (N.H. 1993). When murder is divided into degrees, evidence of some incapacitation due to drugs or intoxication due to alcohol may be introduced to prevent the prosecution from establishing first degree murder. Answer A is incorrect because where murder is divided into degrees, evidence of some incapacitation due to drugs or intoxication due to alcohol may be introduced to prevent the prosecution from establishing first degree murder. Answer C is incorrect because carrying and using a firearm while stoned demonstrates reckless indifferences to a very high risk to human life which meets the legal definition of second degree murder as opposed to manslaughter. Answer D is incorrect because carrying and using a firearm while stoned demonstrates reckless indifferences to a very high risk to human life which meets the legal definition of second degree murder.

A state law made it a misdemeanor offense to drive a vehicle without a driver's license. After driving for 1years, Veronica had her driver's license suspended after a DWI conviction. Nonetheless, she continued to drive. One day, she lost control of her car while changing the radio channel. She veered onto the sidewalk and hit a pedestrian, killing him immediately. The district attorney charged Veronica with homicide. Which of the following is the district attorney's best argument? A: Veronica's violation of the law requiring a driver's license made her guilty based on negligence per se as the law was designed to protect pedestrians from incompetent drivers. B: Veronica created a very unreasonable risk of death when she lost her focus on the road while changing the radio channel. C: Although mere negligence is insufficient to sustain a murder conviction, it is sufficient to sustain a conviction of involuntary manslaughter. D: The pedestrian's death resulted from Veronica's commission of an inherently dangerous misdemeanor.

B: Veronica created a very unreasonable risk of death when she lost her focus on the road while changing the radio channel. Rationale: Answer B is the best answer because involuntary manslaughter is an unintended killing that results from conduct that created a very unreasonable risk of death, or from the commission of a misdemeanor that is inherently dangerous. When a defendant's actions create such a risk, she could be found guilty of involuntary manslaughter. Here, Veronica's driving without a driver's license is a misdemeanor offense that is not inherently dangerous. However, it is a solid argument (and indeed the best among those proffered) that she created a very unreasonable risk of death when she was distracted by changing the radio channel. Answer A is incorrect because negligence per se is a torts concept that is inapplicable in criminal law. Answer C is incorrect because it is inaccurate as mere negligence will not result in a criminal conviction. Answer D is incorrect because driving without a license is not a misdemeanor that is inherently dangerous.

Linda is planning to rob a jewelry store when it received a shipment of diamonds. She bought a gun and a mask as well as cased the jewelry store for how to proceed when the diamonds were delivered. On the day of the robbery, she went to the store and was a block away when she bumped into a high school friend. As a result, she missed the timing of the jewelry delivery. Pursuant to the Model Penal Code, can Linda be found guilty of attempted robbery? A: Yes, because Linda had taken the first step toward robbing the jewelry store when she ran into her friend. B: Yes, because Linda took a substantial step toward robbing the jewelry store. C: No, because Linda had not taken the last step toward robbing the jewelry store when she ran into her friend. D: No, because Linda's actions did not show her unequivocal intent to rob the jewelry store.

B: Yes, because Linda took a substantial step toward robbing the jewelry store. Rationale: Answer B is the best answer because correctly states the Model Penal Code standard of requiring a substantial step. Here, Linda got a gun and a mask as well as planned the robbery. Answer A is wrong because the Model Penal Code does not require that the defendant take the last step toward completing the crime. All that is required is a substantial step, which Linda met when she got a gun and a mask as well as planned the robbery. Answer C is wrong because it does not rely on the Model Penal Code standard, does not require that the defendant take the last step toward completing the crime. All that is required is a substantial step, which Linda met when she got a gun and a mask as well as planned the robbery. Answer D is wrong because it uses the wrong standard as unequivocality is one of the many common law standards.

Patrick lives by himself in a small house. Keenan threatens to burn Patrick's house down while Patrick is at work if Patrick does not help Keenan commit a bank robbery. Assuming that Keenan is ready and able to carry out this threat, does Patrick have a duress defense if he helps Keenan commit a bank robbery? A. Yes, because Keenan has coerced Patrick into helping with the robbery. B. Yes, because Keenan has threatened Patrick with severe harm. C. No, because Keenan's threat does not create a duress defense. D. No, because duress is not a defense to property crimes.

C. No, because Keenan's threat does not create a duress defense. Answer C is the best answer because Keenan's threat does not put Patrick into an imminent such that an ordinary person would have been placed into fear because of it. Answer A is wrong because the word "coercion" presumes the existence of a duress defense. Coercion does not encompass all forms of pressure put on a defendant to commit an offense. Answer B is wrong because severe harm is not the same as specifying that the harm will be physical or death. Answer D is wrong because while duress may be a valid defense to a property crime, there must be a threat of serious bodily injury or death for it to be applicable. Here, there is neither a serious bodily injury or death.

Consider the following factors: I. The infliction of serious bodily harm. II. The intent to commit another serious felony crime. III. The victim is a police officer. IV. The use of a deadly or dangerous weapon. Which of these factors might possibly be included in an aggravated battery statute? A: I, II, and III only. B: I, III, and IV only. C: II, III, and IV only. D: All of the above.

D: All of the above. Rationale: The answer D is the best answer because all four of these aggravating factors might be included in modern aggravated battery statutes. Indeed, the Model Penal Code specifically included factors I and IV as aggravating factors. Specially, section 211.1 (2) addresses Aggravated Assault because the Model Penal Code has abandoned the use of the word battery, but the offense proposed is essentially an aggravated battery based on common law criminal battery. Additionally, some state statutes have included more aggravating factors such as those outlined in II and III.

Laura learned that her husband was having an affair. She followed him to a motel where he was meeting his mistress. When they kissed hello outside their motel room, Laura became upset and pulled out her gun. She only intended to frighten the mistress by shooting over her head. The bullet did not hit the mistress, but went through her blouse sleeve tearing it. With which of these crimes should the prosecutor charge Laura? A: Assault with a deadly weapon. B: Attempted murder. C: Battery. D: Assault with a deadly weapon and battery.

D: Assault with a deadly weapon and battery. Rationale: Answer D is the best answer because Laura has committed both offenses. First, a battery constitutes either a bodily injury or an offensive touching, including contact with the victim's personal effects such as clothing. See generally Adams v. Commonwealth, 53S.E.2d 347 (Va. App. 2000); State v. Gordon, 560 N.W.2d (Iowa 1997). Here, Laura committed a battery on the mistress by tearing her clothing with the bullet. Second, because the offense was committed with a gun, the crime of assault with a deadly weapon is also available consistent with Model Penal Code §211.1. See Russell L. Weaver, et al., Criminal Law A Contemporary Approach 454-5(2d ed. 2014). Answer A is incorrect because although the prosecutor can and should charge Laura with assault with a deadly weapon, she can also be charged with battery. Answer B is incorrect because a defendant cannot be convicted of any criminal attempt, if the defendant lacked the intent to commit the crime itself. Here, Laura cannot be convicted of attempted murder because she had no intent to kill the mistress, but instead was trying to scare her. Answer C is incorrect because although the prosecutor can and should charge Laura with battery, she can also be charged with assault with a deadly weapon.

Ethel decided to kill her best friend Lucy. She sought the help of her boyfriend Fred, who was a doctoral candidate in chemistry, to provide her with ricin. Fred suspected something might be amiss so he gave Ethel penicillin instead of ricin. Ethel put this penicillin in a cocktail that she prepared for Lucy. After she finished drinking it, Lucy had a severe, but fatal allergic reaction to the penicillin. With which offense should the district attorney charge Fred? A: Murder. B: Manslaughter. C: Criminally negligent homicide. D: Fred cannot be prosecuted for any of these offenses.

D: Fred cannot be prosecuted for any of these offenses. Rationale: Answer D is the best answer because Fred cannot be convicted as Ethel's accomplice because there is no evidence that he offered any assistance, advice, or encouragement with the intent to help the principal. Here, Fred specifically gave penicillin with the hope of preventing any foul play such as a homicide. There is no evidence that Fred had the intent to kill Lucy. Answer A is incorrect because murder is the unlawful killing of another with malice aforethought. Here, Fred lacked any intent to kill Lucy. Answer B is incorrect because voluntary manslaughter requires a killing after an adequate provocation and involuntary manslaughter involves a killing in the commission of a felony or certain misdemeanors that are inherently dangerous. Here, those circumstances do not apply to the death of Lucy by the penicillin provided by Fred. Answer C is incorrect because criminally negligent homicide requires that negligence that is greater than a reasonable person standard. Here, there is no evidence that supplying penicillin is unreasonable.

Luther owned a bakery and always took Tuesdays off. One Tuesday, Stan robbed the bakery with a gun. The sales clerk was so upset by the sight of the gun, she gave him everything including the money in the safe. When the sales clerk later called Luther and told him about the robbery, Luther became so upset that he had a heart attack and died. After Stan was caught, he was charged in a jurisdiction that has law establishing the felony murder doctrine. Is Stan guilty of murder? A: Yes, because Stan could foresee that the robbery would result in Luther's death. B: Yes, because robbery is a dangerous felony. C: No, because Stan could not have foreseen a death by heart attack during a robbery. D: No, because the Luther's death did not occur while Stan committed the robbery.

D: No, because the Luther's death did not occur while Stan committed the robbery. Rationale: Answer D is the best answer because the felony murder doctrine only applies when a victim dies during the commission of a felony. Here, Luther suffered the heart attack after the robbery was over. Consequently, Stan's felonious conduct of the robbery happened long before Luther's death happened, and the felony murder doctrine is inapplicable. Answers A and B are incorrect because the felony murder doctrine only applies when a victim dies during the commission of a felony. Here, Luther suffered the heart attack after the robbery was over. Answer C is incorrect because it is foreseeable that a person could die from a heart attack during a robbery, which would implicate the felony murder doctrine. Here, however, the heart attack happened after the felony was over.

Yvonne was at a rally protesting an increase in college tuition. When the police started breaking up the rally, the crowd became agitated. Yvonne fired a bottle rocket over the gathering as a joke, but it went lower that she intended and barely missed an officer's head. However, the officer was looking in the other direction so that she was unaware of the fact that Yvonne had fired the bottle rocket. Nonetheless, Yvonne was charged with assault. Is Yvonne likely to be convicted of assault? A: Yes, because the firing of the bottle rocket was a substantial step. B: Yes, because assault does not require intent. C: No, because the officer did not see the bottle rocket. D: No, because the officer did not see the bottle rocket and Yvonne did not intend to strike the officer.

D: No, because the officer did not see the bottle rocket and Yvonne did not intend to strike the officer. Rationale: Answer D is the best answer because it demonstrates that neither means of establishing a criminal assault has been met. Criminal assault requires either that the defendant attempted to commit a battery or that the defendant intentionally places the victim in reasonable fear of imminent bodily harm. Here, Yvonne had no intention to commit a battery. Moreover, the officer was unaware of the bottle rocket so she was no placed in imminent apprehension. Answer A is incorrect because some overt act is necessary to establish a defendant's intent for an attempt of a criminal offense. Here, however, there was no intent by Yvonne. Answer B is incorrect because assault does not require intent. Answer C is incorrect because it ignores the fact an assault could still have occurred if the officer was placed in reasonable fear of imminent bodily harm.

The purpose of specific deterrence is to prevent an individual from continuing to commit the crime by incapacitating the defendant. True or False

False (p. 23)

When a criminal statute does not contain an explicit mens rea element, the law establishes a strict liability offense. True or False

False (pp. 153-54)

At common law, there were only 4 levels of culpability for mens rea. True or False

False

The year and a day rule applies to homicides pursuant to the Model Penal Code. True or False

False (160)

The conviction in Acosta was reversed because there was no actual cause. True or False

False (166-74)

The conviction in Acosta was reversed because there was no legal cause. True or False

False (166-74)

A defendant may be convicted in a result crime based on actual cause and the necessary mens rea. True or False

False (194-95)

Legal cause is similar to the "but for" principle of causation in tort law. True or False

False (195).

Principals in the second degree are present at the scene of the crime and perpetrate the offense. True or False

False (197)

Merger of principals and accessories involves all four of the distinct common law categories. True or False

False (199)

Mere presence is sufficient to establish accomplice liability. True or False

False (208-09)

At common law, the actual agreement followed a unilateral approach. True or False

False (292)

Mens reas for a conspiracy offense simply requires the intent to agree with another to commit the criminal act. True or False

False (300)

Conspiracy is often harder to prove than attempt because of the need to prove a substantial step. True or False

False (309)

A defendant cannot be convicted of a conspiracy charge based on the inadmissible hearsay statement of a co-conspirator. True or False

False (322)

The prosecution need prove that each defendant personally did a particular overt act in furtherance of the criminal conspiracy. True or False

False (325-330)

The overt act is part of the actus reus of the conspiratorial offense. True or False

False (353)

"... whoever, by force or violence, or by intimidation, takes from the other person ... any thing of value belonging to, or in the possession of, any bank." General Intent or Specific Intent?

General Intent

Co-conspirators are held culpable for the criminal acts of their co-conspirators undertaken in furtherance and within the reasonably foreseeable scope of the conspiracy. True or False

True (344-49)

A defendant may have a legal duty to act based on assumption of care. True or False

True (p. 58)

The common law favored mens rea so courts will often look for evidence that a legislature intended to enact a strict liability offense. True or False

True (pp. 101-09)

Possession requires more than mere control, but must also include knowledge about the control. True or False?

True (pp. 33-37)

Criminal convictions based on conduct as opposed to status generally satisfy the Constitution. True or False

True (pp. 45-52)

William robbed a jewelry store, but the police arrived quickly and surrounded the store. So William took the salesclerk as a hostage. In an attempt to escape, William made the salesclerk wear his mask as well as clothing and go out the front door to distract the police while he fled. Unfortunately, one of the officers thought the salesclerk was not only the robber, but that a weapon was brandished. That officer shot and killed the salesclerk. While that happened William fled the store, but he was apprehended two hours later and charged with murder. What is the most likely reason that William would be convicted of murder? A: William caused the salesclerk's death during the course of a robbery. B: William was still in the store and had not fled when the salesclerk was killed. C: William's forcing the salesclerk to exit the store with his mask and clothing on established homicidal intent. D: The officer improperly used lethal force in killing the salesclerk.

Rationale: Answer A is the best answer because the cashier was killed during the robbery, which is a felony offense, and thus the felony murder doctrine applies. A defendant need not intend to have killed the victim for the doctrine to apply, but the death just has to be a foreseeable consequence of the felony. Here, William sent the salesclerk out wearing his clothing and mask after police officers had surrounded the store to create a diversion so that he could get away. Answer B is incorrect because it does not matter whether William was still in the store when the cashier died. Indeed, even if Alexander left before the salesclerk exited the store, he still would have been culpable pursuant to the felony murder doctrine. Answer C is incorrect because William's decision to put the salesclerk into his mask and clothing before sending her out the front door does not establish that he had the intent for her to die. His actions were recklessly indifferent to her life, but that does not establish the standard for intent in a murder. Answer D is incorrect because the felony murder doctrine does not hinge on whether the police officer used lethal force. Instead, any death that is reasonably foreseeable during the commission of any felony triggers the felony murder doctrine.

"A person commits burglary if without the consent of the owner, the person enters a habitation or building not then open to the public, with the intent to commit a felony, theft, or assault therein." General Intent or Specific Intent?

Specific Intent "with the intent to commit a felony, theft, or assault therein"

"Whoever forcibly assaults any person designated as a federal officer while engaged in the performance of his official duties, with the purpose of impeding the officer's performance of those duties ...." General Intent or Specific Intent?

Specific Intent "with the purpose of impeding the officer's performance of those duties"

A defendant may have a legal duty to act based on the creation of the risk by the defendant. True or False?

True

A defendant's actions may be the cause of death even though absent the defendant's conduct, the death still would have happened. True or False

True (162)

An intervening cause is dependent if it is a normal or involuntary response to a defendant's actions. True or False

True (170)

Tort liability concept of proximate cause has no proper place in prosecutions for homicide as a more direct causal connection is required for conviction. True or False

True (180)

A dependent intervening cause breaks the chain of causation when it is abnormal. True or False

True (195)

An intervening cause breaks causation when the result is too remote or accidental. True or False

True (195)

At common law, the acquittal of the principal barred prosecution of the accessory. True or False

True (198)

In the modern approach, the acquittal of the principal barred prosecution of the accessory. True or False

True (198)

When an indictment alleges that a defendant is a principal, then the defendant is on notice regarding liability as an accessory. True or False

True (200-04)

"When an indictment charges a defendant with acting in concert, then the defendant can be charged as either a principal or as an accessory. True or False"

True (205-06)

Regarding accomplice liability, the intent can be inferred from a person's actions. True or False

True (220-21, 253)

In misdemeanor cases, all accomplices were treated as principals. True or False

True (253)

The mental state for complicity is the purpose of promoting or facilitating the commission of the crime. True or False

True (253)

The Wharton rule occurs when an agreement by two persons to commit a particular crime cannot be prosecuted as a conspiracy because the crime's nature necessarily requires the participation of the two persons. True or False

True (298)

The actus reus of a conspiracy is the agreement among conspirators, which can be established based on circumstantial evidence. True or False

True (314)


Related study sets

Prep U: Chapter 63 Adult Nursing

View Set

Which of the following statements defines the open door policy?

View Set

AD Banker Chapter 2 Exam Questions

View Set

Quiz 7: Thyroid & Parathyroid; Adrenal & Pituitary

View Set

MATH 1530 - Statistics; Chapter 1 & 2

View Set